The Physics of Global Warming

"Global warming is the greatest and most successful pseudoscientific fraud I have seen in my long life." -Hal Lewis

The most valuable natural resources we have at our disposal during our brief lives are the following.

That's right, the Earth and the Sun. And if we want life on Earth to continue as we know it, we have to avoid destroying our own natural environment. The big questions are whether we're actually damaging it to the point of devastating destruction, and if so, what we need to do to fix it.

One of the things we've measured reasonably well -- at, for instance, weather stations all across the world -- is the global average temperature since the late 19th century. Have a look.

As the data shows in no uncertain terms, the world is -- on average -- getting warmer. And it's doing so at a very fast rate: the global average temperature has increased by approximately one degree Celsius in the last century. Three years ago, the American Physical Society released this official statement on climate change:

Emissions of greenhouse gases from human activities are changing the atmosphere in ways that affect the Earth's climate. Greenhouse gases include carbon dioxide as well as methane, nitrous oxide and other gases. They are emitted from fossil fuel combustion and a range of industrial and agricultural processes.

The evidence is incontrovertible: Global warming is occurring.

If no mitigating actions are taken, significant disruptions in the Earth's physical and ecological systems, social systems, security and human health are likely to occur. We must reduce emissions of greenhouse gases beginning now.

Because the complexity of the climate makes accurate prediction difficult, the APS urges an enhanced effort to understand the effects of human activity on the Earth's climate, and to provide the technological options for meeting the climate challenge in the near and longer terms. The APS also urges governments, universities, national laboratories and its membership to support policies and actions that will reduce the emission of greenhouse gases.

There are a few people out there actively, vigorously, and loudly disputing this. Most recently, there's a lot of hubbub over Hal Lewis's statements, such as my quote at the top, that cries of global warming are a hoax, and that there is a scientific fraud being perpetrated on the largest scale of all time. He recently resigned from the American Physical Society over their unwillingness to hold a Topical Group on Climate and Environment, and details his disgust here.

Other people have tried to explain why the science is sound, and I'm going to try to make it as simple as possible.

No, seriously, enough joking around. Let's go directly to the science of what makes the Earth the temperature it is.

This is the Sun.

The Sun emits light. What form of light?

Well, visible light, but also ultraviolet and infrared light. Most of this light comes in through our (mostly) transparent atmosphere, and strikes (and gets absorbed by) the Earth's surface. Without the Sun, the Earth would be about three degrees above absolute zero; thanks to the Sun, we're more like 300 degrees above absolute zero.

But we don't just sit around, absorbing the Sun's energy and getting hotter and hotter. The Earth rotates, and during the night, it radiates this absorbed energy back into space. Although it doesn't radiate visible light, the Earth radiates in the same way the Sun does: like a blackbody.

These two graphs look almost the same! The big differences are written along the axes. The Sun, of course, is much, much hotter and more energetic than the Earth! The Sun emits an intensity of light -- in these units -- that's around 10,000,000 times that of Earth. The Earth, being so far away, only absorbs a tiny fraction of this light.

But, like we said, the Earth also radiates the energy absorbed during the day back into space at night. But it isn't radiating visible light; the other main difference is the wavelength of light that the Earth emits; instead of visible light, this light is far into the infrared.

If this were all that were going on, the Earth would, on a year-to-year average, remain at the same temperature.

But if, hypothetically, if you fill the Earth's atmosphere with gases that absorb the light the Earth emits, you will heat up the Earth. That's what a greenhouse gas is, and the most abundant one on Earth is carbon dioxide, or CO2, which absorbs best at around four microns.

If we look at sunlight, we see that only a tiny, tiny fraction of the sunlight comes in at four microns, meaning that CO2 has a negligible effect on the energy coming in.

But what about the energy leaving?

We have to look at "Earthlight" for that, and wouldn't you know it? A substantial fraction -- somewhere around 1% of the total energy emitted by Earth -- can get absorbed by the CO2 in our atmosphere.

What happens to that energy after the CO2 absorbs it?

It gets re-radiated back towards Earth, heating the planet up again!

In other words, the carbon dioxide acts like a two-way mirror, letting all the sunlight in, but reflecting the emitted heat back towards us! This last part -- the reflection of the outgoing heat -- raises our temperature, the same way reflected sunlight heats things up in a solar cooker.

That's the principle behind global warming. Combine that with the observations that the Earth is getting warmer and the fact that we've added around 1,400 billion metric tons of CO2 to the atmosphere (thanks, stone1343) since the industrial revolution,

and you've got something that's incredibly suggestive, logical, and quantifiable. Now we come to the question being thrown around by global warming deniers: Is it incontrovertibly, scientifically proven that the increase in CO2 causes global warming?

No, no it isn't. But it's unproven in the same way that all scientific theories are unproven; if you wait until every possible conceivable alternative has been explored, tested, and rejected, you will never conclude anything. In this case, where the Earth is at stake, it will likely be too late to undo the damage.

This isn't just a scientific issue; it isn't even an environmental, quality-of-life issue. We are doing a demonstrably destructive thing, by emitting the CO2 we are emitting into the atmosphere. If we don't stop doing it, we will undoubtedly raise the Earth's temperature, and it becomes a question only of "by how much?" And if you want to see the most catastrophic example of what carbon dioxide can do to a planet, look at the closest planet to us: Venus. We're nowhere near that, yet, and I want it to stay that way.

And whether you're a democrat, republican, independent, a global warming denier or a climate change advocate, or anything in between, you want the Earth to stay that way, too, whether you know it or not.

Categories

More like this

One quibble. You wrote:

The Earth rotates, and during the night, it radiates this absorbed energy back into space.

The Earth radiates absorbed energy back into space at all hours of the day, not just at night. There's obviously a net gain of energy during daylight hours and a net loss at night - hence the diurnal temperature cycle.

Except we've added a *lot* more than 10 Gt CO2. It's more like 30 Gt every year. I once did some rough calculations, off the top of my head, 400 ppm corresponds to roughly 3,000 Gt CO2 currently in the atmosphere, so at 300 it would've been 2,250 meaning 750 Gt more now than then. Total emissions of 1,500 Gt, half of which has been absorbed by the oceans and other sinks...

By stone1343 (not verified) on 10 Nov 2010 #permalink

Yes but completely free markets are self-regulating and American and God-endorsed, therefore AGW can't be true. And dozens of people will give you the sameold sameold cockeyed reasons why below.

Two issues here.

First,

'Is it incontrovertibly, scientifically proven that the increase in CO2 causes global warming?

No, no it isn't.'

is totally wrong. It has been proven over and over again that CO2 is a greenhouse gas in that it is a strong absorber of IR light. In fact, we have CO2 lasers based on the exact same physics that describe the CO2-forced greenhouse effect. All of this is based on quantum mechanics, is which arguably the most well-tested theory science has produced so far.

I don't see anyone arguing over the same physics that describe the functioning of their cell phones and LED tvs.

We also know that greenhouse gases lead to a greenhouse effect for the reasons you describe above. So if we know, from quantum mechanics, that CO2 is a greenhouse gas and an increased greenhouse effect increases temperature, then it IS 'incontronvertibly' proven that increased levels of CO2 will increase temperature. ...and this has happened further supporting quantum mechanics and the semi-classical formalism of molecule-light interactions.

Moreover, the more precise question is 'will the increased greenhouse effect due to human emitted CO2 cause other weather effects that affect people's lives?' From there, a political debate can ensue.

Second, the quibble about Venus shows a lack of basic physical understanding of climate.

The single most important aspect of atmospheric effects in terrestrial climate (those effects dealing with greenhouse gases and the greenhouse effect) are due to the Coriolis force. The Coriolis force is why we have jet streams and Hadley cells.

On Venus, however, there are less than two days per year. In fact, the days change due to Venus' orbit around the sun before they change due to the rotation of the planet itself. This implies that the Coriolis force is much smaller than on earth. Two orders of magnitude smaller. So almost all the weather (and therefore climate) on Venus is due to the fact that the sun is continuously heating one side of the planet, but not the other. That temperature difference creates substantially different weather patterns than we find on earth.

Because of this fact, it's not appropriate to compare the greenhouse effect on Venus to that of earth. It's a common misunderstanding unfortunately.

One important physical aspect that many physicists seem to miss in context to global warming is the fact that because more heat is being trapped in the lower troposphere, the higher layers of the atmosphere get colder. This creates a greater difference in temperature between the lower and higher layers of the atmosphere. Under these conditions, there should be higher rates of energy transfer between the lower and higher layers of the atmosphere, according to the second law of thermodynamics. Obviously that transfer cannot be radiative, but luckily, the atmosphere has other ways to transfer heat away from the surface of the earth.

It will be interesting to see how these two seemingly competing physical processes will affect our world in the future.

Question: Is my assumption that the re-radiation of the energy absorbed by the atmosphere is omni-directional wrong? seeing that you say:

In other words, the carbon dioxide acts like a two-way mirror, letting all the sunlight in, but reflecting the emitted heat back towards us
By Who Cares (not verified) on 10 Nov 2010 #permalink

First of all equating warming with damage is absurd and completely unscientific. Many regions and species would benefit from warmer climate, yes, there would be losers too, but that is how it always has been and always will be, the climate never was and never will be stable, with or without humans.

As for the anthropogenic global warming hypothesis the major problem with it is that there is no way to tell what part of recent warming is due to anthropogenic emissions and what part is due to natural changes of the climate. Correlation is not causation, the CO2 we emit may be responsible for only a tiny fraction of the warming, we don't have any solid data which could be used to reliably establish the strength of the effect of our emissions, all we have are untested speculations.

You are also extremely dishonest when you say that the issue "is unproven in the same way that all scientific theories are unproven." This is blatant nonsense, most accepted scientific theories have been tested in countless carefully controlled laboratory experiments, unfortunately such controlled experiments are simply impossible when it comes to global climate. This alone greatly undermines the credibility of climate science when compared to disciplines like physics, chemistry or biology.

As a general rule the credibility of each scientific discipline is directly proportional to the ease of preforming and interpreting experiments in it. In this regard climate science is even worse then economics and we all know how well global economic models work.

Global climate is an extremely complex chaotic system with great many feedbacks and there is absolutely no reason to believe that we even begin to understand all the relevant elements affecting global temperature. So far not a single computer climate model managed to correctly postdict the climate of the past century, which is really the most obvious and basic sanity check one can imagine, until we have models which pass such verification all the climate modeling is pure GIGO - garbage in garbage out.

So while our planet is certainly important, from a scientific point of view anthropogenic global warming is nothing more then a plausible hypothesis.

Dang it Dave beat me too it. Ethan (or anyone who knows the answer) could you post a quick and dirty rundown on why the absorption spectra is what it is. I saw the explanation once at climate progress I think but did not read it and can't find it again. I tried to search it but the internet is a scary place.

By beardedbeard (not verified) on 10 Nov 2010 #permalink

Styx,

get a clue.

'This is blatant nonsense, most accepted scientific theories have been tested in countless carefully controlled laboratory experiments, unfortunately such controlled experiments are simply impossible when it comes to global climate. This alone greatly undermines the credibility of climate science when compared to disciplines like physics, chemistry or biology. '

Talk about blatant nonsense.

As I already pointed out, quantum mechanics, the most well-tested theory science has yet produced, explicitly predicts that CO2 molecules will absorb IR light emitted by warm terrestrial surface. As you increase the concentration of CO2 molecules, the absorption of IR increases linearly, as per the Beer-Lambert law. A law that has been tested in labs all over the world for the last century to explain the vast majority of variation we measure in the absorption of light by molecules.

Because some of that energy absorbed by an increasing concentration of CO2 molecules is re-emitted back toward the surface of the earth, energy builds up in the lower troposphere. That causes the temperature to increase, as per the greenhouse effect that has been predicted, modeled and tested in the real world for almost 200 years since the time of Fourier!

More specific to your actual claim, climate science is physics. It is the application of very basic physical principles to the climate system. There is no distinction in the actual science.

If you're going to pretend to know what you're talking about, at least try not to make a complete idiot out of yourself.

Styx, you say...

First of all equating warming with damage is absurd and completely unscientific. Many regions and species would benefit from warmer climate, yes, there would be losers too, but that is how it always has been and always will be, the climate never was and never will be stable, with or without humans.

Well, I guess if you happen to be a hyperthermophile living in a Yellowstone geyser that might be somewhat true. However for the vast majority of species it would certainly change their environment and if you think the climate is complex and hard to model, you might want to consider that the interactions and feedback loops of multiple global ecosystems being impacted by a changing, not necessarily just warming climate, which might include droughts or excessive rainfall or changes in ocean currents and ocean chemistry etc... might be a pandora's box we might not want to open without a little more knowledge as to the consequences.

Just for shits and giggles you might want to take 24 minutes out of your busy schedule and watch this talk on Saving Life on Earth by E.O. Wilson. http://www.ted.com/talks/e_o_wilson_on_saving_life_on_earth.html

While it is also true that the climate never was and never will be stable, with or without humans, I at least would prefer to apply the precautionary principle and err on the side of minimizing the possibility of sending the fewest number of currently still stable parameters past their tipping points into new and unknown territory... the beauty of strange attractors in chaos theory notwithstanding.

My personal feeling is that we somehow need to transition to a new global economic paradigm based on biophysical steady state economics and we need to do this sooner rather than later but then I'm just a great ape and not a hyperthermophile and I depend on a much more complex ecosystem for my well being than a simple volcanic hot spring.

If you think you agree with my last statement please consider visiting http://steadystate.org/

By Fred Magyar (not verified) on 10 Nov 2010 #permalink

Maxwell thanks for (accidentally) answering my question while rebutting Styx.

Also am I correct in the following line of reasoning ?
All I need is the ability to collect data on the spectrum received by the earth (using for example a photometer). To double check stick a few satellites in orbit that do the same (so I can check if incoming matches staying + outgoing)
Put an overlay of the radiation emission pattern of CO2 over this data received from the photometers and I can see how much of the radiation received by the earth comes from CO2 absorption and re-radiation.

By Who Cares (not verified) on 10 Nov 2010 #permalink

Even if you accept that there is a significant chance of AGW, then the issue is economic. How much do we owe the future? Since economists can't even agree on what the discount rate is i.e. how much is a dollar worth in 10 years it becomes impossible to work those numbers. What is needed is to estimate the costs occuring when if AGW happens, and the costs to mitigate/eliminate. The as a society we decide if we want to buy that insurance policy. Just like if you live on the Gulf Coast and buy a place free and clear you can decide if you want hurricane insurance.
But as I say the current value of the costs of AGW can not be determined since we can't agree on a discount rate so we can't even start the necessary conversation.

It seems like the liberal tree-hugging crowd that are so worried about global warming are also the same people who are in denial about overpopulation being the basic cause of resource depletion, such as the growing shortage of air that has been scrubbed of excess CO2 by rain forests and such. Using less energy is difficult. Curtailing the exploding population of the U.S. by ending immigration and migration of fecund illegal aliens with a 3.5 fertility rate who come here and drive their jackpot babies around in enormous Chevy Suburbans is trivially easy. So who's really in denial?

Just some suggestions:

1. Superimpose the blackbody curve for the sun and the one for the earth; of course you will need to scale the one for the earth so it doesn't look too flat, but you can put the scale factor into the plot - or plot it twice (unscaled, scaled).

2. The earth radiates with peak energy somewhere closer to 10 microns and CO2 has absorption bands elsewhere, so the ~4.5 micron absorption band is only part of the story. Unfortunately my current computer is not set up with the radiative transfer code so I can't run models to show things better; otherwise I could replace that scan of a plot from the QJRMS with something much nicer to look at. It would also be possible to show the changes from the current 388ppm in the atmosphere and the magic number of 560ppm which the IPCC seems obsessed with. These will be transmission spectra though, which assumes a hot source and negligible emission of radiation by the gases - so they are only indicative. It takes a lot more time to set up a full radiative transfer calculation which takes into account the absorption and emission throughout the atmosphere, which is dependent on the temperature of the gas and would make much less sense to a non-expert.

@Who Cares: In principle you need to measure the incoming solar radiation at the top of the atmosphere, and what the earth is emitting and reflecting at the top of the atmosphere (emissions will be measurable day and night). You don't need measurements at the ground at all - at least in principle. The measurements made from orbiting instruments would be sufficient to determine the imbalance in energy. Now if this measurement is precise enough then we would know how much more energy is going into the earth system. Unfortunately reality is far more difficult - we don't have the coverage and we don't have the precision on measurements, so we do need extensive measurements on the ground as well to help constrain things. There are a few exploratory satellite instruments being proposed at the moment which would address a number of issues (and maybe lead to the deployment of more such instruments).

Just to complicate things even more, knowing how much extra energy we're taking in is only part of the problem. We also need to get some idea of how that energy is being redistributed around the globe - for example, how much goes into warming various parts of the atmosphere, melting ice, warming the oceans, etc.

By MadScientist (not verified) on 10 Nov 2010 #permalink

@beardedbeard:

"... could you post a quick and dirty rundown on why the absorption spectra is what it is"

What do you mean by that? The absorption spectrum is simply what we have observed with instruments. In this particular instance it is due to changes in the energy of the CO2 molecule which are associated with changes in the rotation of the molecule as well as with changes in the mean distance of atoms in the molecule (called changes in the "vibration" state). The details of the shape of the absorption lines, their relative strengths, and so on literally fills books, so I think you need to refine your question.

By MadScientist (not verified) on 10 Nov 2010 #permalink

It seems like the liberal tree-hugging crowd that are so worried about global warming are also the same people who are in denial about overpopulation being the basic cause of resource depletion, such as the growing shortage of air that has been scrubbed of excess CO2 by rain forests and such. Using less energy is difficult. Curtailing the exploding population of the U.S. by ending immigration and migration of fecund illegal aliens with a 3.5 fertility rate who come here and drive their jackpot babies around in enormous Chevy Suburbans is trivially easy. So who's really in denial?

Quite the elaborate strawman you've erected there. Who says liberals are in favor of overpopulation? If anything, they're the ones fighting it by encouraging birth control while religious conservatives decry such things for allegedly being anti-human. And immigration is a completely separate red herring of an issue. Even if we do as you say and stop all immigrants from coming here, that in and of itself will do precisely squat with respect to rectifying AGW.

re @AL comment on @Luther
I think the phrase "come here and drive their jackpot babies around in enormous Chevy Suburbans" is the clue. Population growth is fine if it only happens in poor countries where per capita emmissions are a tiny fraction of those in the US, so keeping immigrants out does reduce global emmissiions. Perhaps US foreign policy should be targeted at keeping poor countries poor, or maybe it already is....

Ethan:

"you want the Earth to stay that way, too, whether you know it or not."

Luther:

"the liberal tree-hugging crowd"

Ethan, it would be nice to believe you, but so far the evidence...

I think the phrase "come here and drive their jackpot babies around in enormous Chevy Suburbans" is the clue. Population growth is fine if it only happens in poor countries where per capita emmissions are a tiny fraction of those in the US, so keeping immigrants out does reduce global emmissiions.

Yes, per capita emissions are higher among the more affluent, and immigrants tend to become more affluent else they wouldn't have immigrated to begin with. But then the issue here is not immigration, it is that wealth leads to consumption, and people overseas can still increase their consumption rates when overseas economies grow, as we are seeing in China and India for example. So immigration is still a red herring issue with no real relevance to AGW.

Global Warming is not a fraud. This is really the real thing. We can even feel the effect of it. Climate changes and rising of sea levels are among the few. But still we have time to save what is left. It is not too late. Thanks for sharing your insights.

NO! Climate Change is NOT about pollution!!!!!
Climate Change says CO2 "will" have consequences ranging from unstoppable warming to virtually no effects at all. Now no wonder the lab coat consultants all agree.
Remove the CO2 factor from the equation and what do you have? You have a half century of productive environmentalism and courage to face the future.
You flat earthers and witch burners of climate change will be cursed in the history books.

As for the anthropogenic global warming hypothesis the major problem with it is that there is no way to tell what part of recent warming is due to anthropogenic emissions and what part is due to natural changes of the climate. Correlation is not causation, the CO2 we emit may be responsible for only a tiny fraction of the warming, we don't have any solid data which could be used to reliably establish the strength of the effect of our emissions, all we have are untested speculations.

Nonesense. We can quantify the contributions of the various forcings to quite a high degree of accuracy. Climate (at the global scale) doesn't change without something forcing it, because of the First Law of Thermodynamics. A first-order calculation of the expected forcing from CO2 (based on the Stefan-Boltzmann Law) is actually fairly straightforward and was first performed by Svante Arrhenius in 1896. Unless you have some objection to the Stefan-Boltzmann Law, the First Law of Thermodynamics, or the principles of calculus, there is no grounds for objection to this. The predictions of theory match very well to observations.

Copernicus, Galileo, Darwin, Einstein, Marie Curie, Aristotle, Newton, Darwin, Kepler, Heisenberg, Hubble, Feynman, Linnaeus, Oppenheimer, Archimedes ............Gore? YES Gore, and his political lab coat consultants, promising to lower the seas with taxes and make the weather colder with MY personal sacrifice? WHAT CENTURY IS THIS?
Why were CO2 levels STILL going up while we Human monkeys where emitting less thanks to years of the âless productiveâ âWORLD economy? And do you ever wonder why there were thousands and thousands and thousands more consensus scientists than there were protesters? Why didnât these all knowing scientists march too? And wouldnât they as saintly scientists have declared a planetary emergency and started marching en mass to the UN? We didnât even know these peoples names but that was good enough to condemn our very own children to death by CO2?
How many climate scientists does it take to change a light bulb? None. But they DO have consensus that it WILL change. All kidding aside, sort of, the theory they agreed upon was that the effects could be nothing or deadly. I smell a free pass here folks. And a lot of them agreed with this PREDICTION and of course those who didnât agree were just evil, neocon, pollution lovers. Now thatâs funny! Meanwhile, the UN had allowed carbon trading to trump 3rd world fresh water relief, starvation rescue and 3rd world education for just over 24 years of climate control instead of population control. Nice Job!
Climate scientists were to science, politics and journalism what abusive priests are to religion. THIS was another Bernie Madoff. This was another WMD. This was another Iraq War of lies and fear mongering and climate WMDâs. Scientists not only polluted the planet with their chemicals, they also produced cruise missiles, cancer causing chemicals, land mine technology, nuclear weapons, germ warfare, strip mining technology, deep sea drilling technology and now climate change. I can't do this anymore. Iâm liberal. Iâm a progressive and a Green Climate Change Denier.

By Meme Mine (not verified) on 10 Nov 2010 #permalink

maxwell: "As I already pointed out, quantum mechanics, the most well-tested theory science has yet produced, explicitly predicts that CO2 molecules will absorb IR light emitted by warm terrestrial surface."

Sigh, did I dispute it anywhere? No.

The global climate is not some simplistic linear system it is extremely complex chaotic and nonlinear, there is no reliable way of translating forcing increase due to extra CO2 into global temperature rise. Look into glacial record and you will see plenty of short (on geological timescale = thousands of years) periods when there was no correlation or inverse correlation between CO2 changes and temperature changes. The idea that you can just guess some number and proclaim it the "right" climate sensitivity without any experimental verification is just absurd.

No experimental confirmation - no credibility.

maxwell: "More specific to your actual claim, climate science is physics. It is the application of very basic physical principles to the climate system. There is no distinction in the actual science. "

No, climate science is not physicis, if you don't even understand this basic point it's your problem. Here, see this cartoon, maybe you will learn something
http://www.xkcd.com/793/

Fred: "Well, I guess if you happen to be a hyperthermophile living in a Yellowstone geyser that might be somewhat true... "

Huh? A few degree rise in global temperature would just shift climate zones away from the equator a bit.

Fred: "While it is also true that the climate never was and never will be stable, with or without humans, I at least would prefer to apply the precautionary principle..."

I would also prefer to apply precautionary principle but unfortunately energy is way to vital to human lives to give it up just to be sure. As for "green energy" replacing fossil fuels anytime soon the scenario is completely impractical, for many reasons, some of them listed here:
http://johncarlosbaez.wordpress.com/2010/10/05/power-density/
(which is a pro-AGW blog BTW but heavy on facts and short on hype unlike most others)

@#21

You flat earthers and witch burners of climate change will be cursed in the history books.

Oh how horrible to be cursed by history after we are dead and past caring! If we are wrong, so be it, at least we would go to our graves thinking we were doing the right thing. In the end we will be forgotten, just as you and everyone else will be.

Quit being so silly and bombastic.

Meme mine,

A most appropriate moniker...

promising to lower the seas with taxes and make the weather colder with MY personal sacrifice? WHAT CENTURY IS THIS?

This is the century when it has already become patently obvious that the economic paradigm which supports YOUR notion that you can live the way YOU do and take NO personal responsibility as to the consequences of YOUR unfair exploitation of the resources of the commons, comes to a screeching halt! We have met the enemy and she is us.

Personal sacrifice?! Boo hoo hoo! Have you ever traveled around the world? Have you ever examined the data and done a comparison as to what the average citizen of say the USA consumes in resoureces and wastes in energy compared to say the average citizen of even a demonstrably wealthy country such as Sweden? I won't even suggest a comparison to the citizens of any developing country, or god forbid the billion plus humans living on the edge of subsistance and starvation.

To be clear, I do agree that global population growth must end and it will, because there are physical limits to growth which we are now begining to see quite clearly but to suggest that the poor whiny spoiled and clueless rich children of the west have any right to complain about their plight, and their need to make personal sacrifices, is beyond disingenous... Some of them, it seems, need to be hit upside the head, hard, with a 2X4 as a wake up call!

We need to rebuild our economic paradigm around thermoeconomic and biophysical economic principles.
THIS IS THE 21st CENTURY and you are NO LONGER entitled to hog the resource pie, either make changes yourself, or nature will make them for you whether you like it or not.

By Fred Magyar (not verified) on 11 Nov 2010 #permalink

Oh and I apologise to Ethan and the commenters for my first post being such an unproductive comment. Just like to say, I am certainly not a physicist but I enjoy treading this blog regularly, very informative and entertaining.

Styx,

The global climate is not some simplistic linear system it is extremely complex chaotic and nonlinear,

From the perspective of changes in global temperature over the timescale of a century or two, the global climate is a rather linear system with a bit of stochastic noise superimposed - barring a few specific scenarios like a thermohaline circulation collapse. Chaos in the microstate of the system doesn't imply that statistical aggregate observables behave nonlinearly. And on century scales there really are very few nonlinearities that can't be subsumed into, say, a different effective system feedback in a linearized system.

there is no reliable way of translating forcing increase due to extra CO2 into global temperature rise.

This claim is unsupported by any physics or data.

Look into glacial record and you will see plenty of short (on geological timescale = thousands of years) periods when there was no correlation or inverse correlation between CO2 changes and temperature changes.

And this is a bait and switch. You've switched from century scale climate prediction to millennial scale, and to periods of time with very different boundary conditions (ice sheet configurations and the accompanying albedos, and orbital forcings).

The overall glacial-interglacial cycle is actually one of the strongest links between CO2 and temperature changes. But you have to realize that most of the temperature change in the glacial-interglacial cycle comes from ice albedo, not direct CO2 forcing. (Certainly the CO2 forcing is instrumental in influencing the albedo changes, but that's a separate question of where the actual temperature change comes from.)

Ice sheets have thousand to ten thousand year "memories" of their history. It is not hard to see from even extremely simple models that small perturbations in orbital forcing or even natural variability (in, e.g., the precise location of the boundary of an ice sheet) are sufficient to drive such transient "mismatches" in temperature and CO2. This doesn't somehow erase the ability to determine a climate sensitivity from glacial-interglacial cycles.

This is a far cry from the situation today, where all the massive continental ice sheets are gone from the northern hemisphere, and we're talking about the climate response over the next century or two, resulting from a MUCH larger CO2 forcing.

Paleoclimatologists don't actually agree with your position that the glacial-interglacial cycles render it impossible to estimate climate sensitivity. (You do have to factor in the boundary conditions to realize that a climate sensitivity during a glacial period with lots of ice isn't the same as a modern climate sensitivity, and correct for that.) You might want to reflect on why they don't agree with your position, before making this claim.

The idea that you can just guess some number and proclaim it the "right" climate sensitivity without any experimental verification is just absurd.

Another strawman. Climate sensitivity estimates aren't random guesses unsupported by any empirical data. And they're not single numbers either, but ranges that are compatible with what we know about the climate. They're a consilience of physical calculation and observational data, and a number of different lines of evidence all give compatible ranges of estimates.

No experimental confirmation - no credibility.

Oh ho, now we're on to "physical predictions have no credibility".

Tritely, you can't "confirm" a prediction without waiting for it to come to pass. That doesn't mean that all physical reductions are "absurd". What is absurd is claiming that we don't know anything about physics and the only way to "credibly" estimate climate sensitivity is to double CO2 and wait a really long time to see what happens.

No, climate science is not physicis,

Climate science largely is physics, coupled to some biogeochemistry. Atmospheric-ocean dynamics, radiative transfer, thermodynamics, ice mechanics, ... those are all physics. I don't know what is the point of denying this, except to possibly imply that it's an unscientific enterprise (by being unphysical, or overly "complex", or whatnot).

Huh? A few degree rise in global temperature would just shift climate zones away from the equator a bit.

Sure, there's hyperbole in someone claiming that global warming is only favorable to hyperthermophiles. But a few degrees fall in global temperature ... is halfway to an ice age. You can't just dismiss changes of that magnitude as "shifting climate zones around a bit" as if they're of little consequence.

We also have few or no examples of the climate warming at not just the magnitude but the rate it is projected to, starting from an already-warm state. And we have no examples of what such climate changes do to ecosystems already under massive influence from other anthropogenic activity (land use/land cover change, barriers to migration, pollution, overexploitation, overall habitat and biodiversity loss).

"Ho hum, climate always changes, nothing new here" rings rather false.

As for "green energy" replacing fossil fuels anytime soon the scenario is completely impractical,

Another strawman. The goal isn't to replace fossil fuels overnight, but to slow their consumption and reduce the total amount consumed over the next few centuries.

(which is a pro-AGW blog

And finish it off with a dose of partisanship. There isn't "pro-AGW", there is just supported by science, or not.

Styx, Hyperthermophiles, at least, seem to understand sarcasm.

Re: A few degree rise in global temperature would just shift climate zones away from the equator a bit.

Did you listen to E.O. Wilson's talk? You need to connect a few more dots before you start seeing the big picture...

I would also prefer to apply precautionary principle but unfortunately energy is way to vital to human lives to give it up just to be sure. As for "green energy" replacing fossil fuels anytime soon the scenario is completely impractical, for many reasons, some of them listed here...

I actually own a solar energy company and have been reading and posting on theoildrum.com for over 3 years, I'm well aware of the energy density of fossil fuels as compared to other energy sources. I don't think this is the best place to get into an extensive discussion on that topic, suffice it to say the link you provide is steeped in BAU think. BAU is dead, what comes next is anybodys guess but one thing is for sure, the future ain't gonna be like the past... I was recently in Germany and had a chance to read the Peak Oil Report released to mainstrean media by the Bundeswehr, if you can read German google it and read it...

What is impractical is continuing on our current path over the energy cliff.

By Fred Magyar (not verified) on 11 Nov 2010 #permalink

The comic captures the essence of the issue perfectly. Even if somehow all the experts on climate in the world are somehow completely wrong, are goals to create a more sustainable, cleaner energy infrastructure on the Earth not worth it in their own right?

This is a marketing issue, not a science issue. Just pretend you're not fighting global warming, but you're trying to uh...save kittens from drowning or something. DO YOU HATE KITTENS?

beardedbeard,

the absorption of light is due to the structure of materials. In the case of the greenhouse effect, we're talking about the structure of molecules. According to quantum mechanics, on the scale of a single molecule, the structure is made up of states of discrete amounts of energy. For a polyatomic molecule like CO2, we have to consider each degree of freedom of the molecule in figuring out what states of discrete energy it will have.

In it's own frame, a CO2 molecule has electrons flying around nuclei. The electrons are dealt with on their own as one degree of freedom. The nuclei have two degrees of freedom, however. They can vibrate like balls connected by springs and they can rotate. Both of these 'motions' are quantized, in the sense that there are discrete states of energy that the molecule can populate.

When light is absorbed by a molecule, it means that the energy embedded in the light is of the same energy as the DIFFERENCE between two states in the same degree of freedom. That is, a molecule can transition between two DIFFERENT electronic states or two different vibrational states or two different rotational states. In the specific case of atmospheric CO2, the light absorbed corresponds to transitions in the vibrational states of the molecule.

Given the 'force' of the double bond in a CO2 molecule and specifics about a CO2 molecule's symmetry, we could get a pretty good calculation of the energy of such vibrational transitions. We can also go into a lab and test the predictions of such calculations using lamps or lasers to see where a sample of CO2 molecule will absorb and other more esoteric aspect of light absorption like boardening and lifetimes.

Hope that clears things up a bit.

Who Cares,

it seems like your plan should work. In fact, NASA has done just about the same thing. The problems come in our ability to precisely measure the upward and downward flux of IR light, as you're trying to do.

As I pointed out to Ethan, many people don't know that the earth will 'try' to balance the amount of energy coming in to that going out. That is, as the surface gets warmer, the higher elevations get cooler.

Conservation of energy is pretty important after all.

But that change is not instantaneous. So as the surface gets warmer, the higher elevations can't cool right away and there should be a difference in the amount of IR light being emitted by the atmosphere back out to space.

But the instruments we use to measure the incoming and outgoing flux of energy have an error of about 1% of the flux itself. Many people believe that if we were to precisely measure the effect of an increased greenhouse effect due to human emitted CO2, such an effect would look like a 1% decrease of the total outgoing flux of IR light. So we're looking for a signal that is 1% of the total and our error is at least 1% of the total, so we're stuck on that front until we can get better precision measurements in space of the IR light emitted by the atmosphere.

If you want to measure the downwelling IR light on the surface, you'd just need an IR thermometer. Roy Spencer, of all people, has some very interesting posts on quantitatively measuring the amount of IR emitted back at the surface of the earth from the atmosphere, proving beyond a shadow of a doubt that the greenhouse effect is real.

Take care.

I've had too many such discussions in the past so the amount of time and energy I am willing to commit to them now is rather limited.

Anyway here are some answers to FB.

Me: "The global climate is not some simplistic linear system it is extremely complex chaotic and nonlinear,"

FB: "From the perspective of changes in global temperature over the timescale of a century or two, the global climate is a rather linear system with a bit of stochastic noise superimposed..."

This is just wishful thinking, again see here http://www.xkcd.com/793/

Me: "there is no reliable way of translating forcing increase due to extra CO2 into global temperature rise."

FB: "This claim is unsupported by any physics or data."

The burden of proof is on those who claim it can be done.

Me: **"Look into glacial record and you will see plenty of short (on geological timescale = thousands of years) periods when there was no correlation or inverse correlation between CO2 changes and temperature changes."

FB: *"And this is a bait and switch. You've switched from century scale climate prediction to millennial scale, and to periods of time with very different boundary conditions"

Haha, you do know that centuries are contained in millennia, right? If there are millennia with CO2/temp correlations all over the scale then it goes without saying that there are centuries with those correlations all over the place.
Also the fact that there are different boundary conditions is the whole crux of my argument - different conditions lead to different CO2/temp correlation so it's NOT a simplistic "increase in CO2 = more forcing = warmer Earth" as the person I responded to claimed, quite often in the past increase in CO2 were paired with decrease in temperature.

FB cont.: *"The overall glacial-interglacial cycle is actually one of the strongest links between CO2 and temperature changes. But you have to realize that most of the temperature change in the glacial-interglacial cycle comes from ice albedo, not direct CO2 forcing....This is a far cry from the situation today, where all the massive continental ice sheets are gone from the northern hemisphere..."

If you actually looked at the ice core temps/CO2 graphs I mentioned you would see that the correlation is most unpredictable in late interglacial periods just like the one we have today - no massive continental ice sheets.

FB: "Paleoclimatologists don't actually agree with your position that the glacial-interglacial cycles render it impossible to estimate climate sensitivity..."

Straw man, my position is not that it is impossible to guesstimate sensitivity, but that there is no way to experimentally verify which guesstimate - and different people come up with very different ones - is actually correct now.

Me: "No experimental confirmation - no credibility."

FB: "Oh ho, now we're on to "physical predictions have no credibility".

Huh? Do you even understand scientific method? Experiment is the sole reason science is successful, without experiment to confirm or refute hypotheses science cannot advance and is forever stuck in philosophizing where everyone has their own theory.

And so on...

Repeatedly citing the same XKCD comic does not actually refute the fact that, at the very largest scales, global heat content is simply a question of radiative equilibrium.

Scientists have already done the difficult work.

Miracles will not help; technology will not help; more information will tell us only if we are already too late.

Those who believe only what you already "know"; I suggest you invest in property a few feet above sea level.

Politicians pay attention! Act before its too late.

Styx: A few degree rise in global temperature would just shift climate zones away from the equator a bit.

And shall the hundreds of millions of people who already live in those areas, with climate-sensitive agricultural and population centers, "just shift" to follow the favorable climate? Instead of calling them hundreds of millions of refugees, should we call them "just shifters"? What if the new location of the zone they can function in is otherwise uninhabitable--say, a mountain range? There's a reason why they didn't live there in the first place. If the new proper climate for raising crops shifts into what had once been a heavily urbanized area--well, sorry, but you can't "just shift" wheat into growing through cement. And that's not even to get into the wild ecosystems that may be constrained by natural (mountains, lakes) and artificial (fences, highways, dams, cities) barriers and are un-"just-shift"-able.

The world didn't begin the day you were born--it took a long time to establish human civilization, even longer to establish ecosystems, and it is folly to imagine that any rapid significant change would result in anything other than widespread chaos and death.

And let's not forget deserts. Even if you "just shift the climate bands" enough to place the optimum regional conditions for agriculture into what are now the Sahara, Gobi, and Arabian deserts, "just shift"-ing atmospheric temperature would not be enough to create topsoil. That would take centuries. People gotta eat.

Styx,

This is just wishful thinking, again see here

I hope you realize that comics do not constitute scientific arguments. Nor is your comic even relevant to what is being discussed, since this is a discussion about climate science, not of physicists wandering into other people's fields. Climate scientists themselves disagree with your claims.

Frankly, there is absolutely no evidence of high nonlinearity in the global average temperature response on century scales, in modern-type interglacial climates â either from physics or data (modern or paleo) &mdash.

By "high nonlinearity" I mean of the type that would be inconsistent with linearized dynamics (maybe with more than one time scale) modulated by small interannual to multidecadal stochastic fluctuations (weather noise, ocean cycles, etc.). By "small" I mean relative to the average global temperature change projected over the next century or two.

If you feel there is such evidence, present it. Otherwise it's wishful thinking.

The burden of proof is on those who claim it can be done.

What "burden of proof"? There is an enormous body of evidence linking CO2 to temperature changes through deep to shallow paleoclimate into historical times. You can hardly be unaware of it (but if you are, I'd start by watching Richard Alley's AGU 2009 keynote talk on the web), so I assume that you simply reject all of it as demonstrating anything about the climate.

Haha, you do know that centuries are contained in millennia, right? If there are millennia with CO2/temp correlations all over the scale then it goes without saying that there are centuries with those correlations all over the place.

You've missed the point. Millennial-scale slips in the glacial cycle are linked to millennial-scale processes (ice sheet dynamics) which are not present today.

Also the fact that there are different boundary conditions is the whole crux of my argument - different conditions lead to different CO2/temp correlation so it's NOT a simplistic "increase in CO2 = more forcing = warmer Earth" as the person I responded to claimed, quite often in the past increase in CO2 were paired with decrease in temperature.

An argument for other factors than CO2 influencing climate is not an argument against CO2 having an influence on temperature in general, or against paleo estimates of climate sensitivity specifically.

If you actually looked at the ice core temps/CO2 graphs I mentioned you would see that the correlation is most unpredictable in late interglacial periods just like the one we have today - no massive continental ice sheets.

I was indeed talking about the temperature/CO2 "slips" in glacial periods. If you're talking interglacials, quasi-equilibrium interglacial temperatures are in fact well correlated with interglacial CO2 concentrations, which explains much of the variation in interglacial temperature. Orbital forcing variations account for most of the rest. The time series themselves don't always line up exactly, but much of this is attributable to differences in the oxygen and carbon isotope chronologies.

I don't claim that every fluctuation in CO2 and temperature has been accounted for with respect to known ice volume data and orbital forcings. In fact, I am sure that there never will be any exact accounting, due to data constraints and an inability to construct the details of natural variability.

I do claim that attempts to explain the glacial-interglacial cycles and the diversity between different glacial cycles will fail miserably without including CO2 as a driver, and a climate which is sensitive to CO2 within the published range of climate sensitivity estimates.

Straw man, my position is not that it is impossible to guesstimate sensitivity, but that there is no way to experimentally verify which guesstimate - and different people come up with very different ones - is actually correct now.

As I said myself, there is no experimental way to "verify" what the equilibrium response to doubled CO2 is without actually doubling the CO2 concentration of the planet and waiting. Your argument appears to be that this therefore means that estimates have no "credibility", which is nonsense.

As for different people coming up with different sensitivities, I already said that there is no "one estimate" of climate sensitivity. Because the data are coarse and sparse, there is a range of allowed sensitivities, and we may never narrow that range based on paleoclimate evidence alone. This, again, is different from claiming that the currently allowed range of sensitivities is not credible.

If you want to concede that the current range of estimates is credible, in the sense that if we could run the experiment we'd likely find a result in this range, fine. All I'm saying is that the data do legitimately constrain the allowed sensitivities, and that the current range is credible.

Huh? Do you even understand scientific method? Experiment is the sole reason science is successful, without experiment to confirm or refute hypotheses science cannot advance and is forever stuck in philosophizing where everyone has their own theory.

You apparently do not understand the scientific method yourself, because your representation of it sounds like something you'd hear from an 8th grade textbook, not a practicing scientist. In particular, you don't seem to think that observational sciences constitute credible science (which would include much of geology, astronomy, evolutionary biology, etc.).

What matters to science is not controlled experiments per se, although those are great when available. What matters is theoretical frameworks that are consistent with the body of empirical evidence. You can't go back in time and re-run a glacial-interglacial cycle, but you can test which hypotheses about such cycles are compatible with the collection of geologic and biologic data available, as well as with modern observations and tested physical laws.

FB,

I'd say you're wasting your time continuing to refute an argument largely based on a misrepresentation of a cartoon.

What does Ethan think?

Nothing could possibly be bad about climates changing! It isn't like plants will react differently to the larger amounts of CO2 in the atmosphere or anything.

http://blog.deborahblum.com/?p=620

By Lynxreign (not verified) on 11 Nov 2010 #permalink

@MadScientist & Maxwell:
I know that what I suggested was extremely simplistic (to the point of almost being a parody of that XKCD comic). So thank you both for explaining (some of) the extra hurdles on the way to get the real data needed.

Styx you make the following claim:
quite often in the past increase in CO2 were paired with decrease in temperature.

Could you provide evidence (as per your demand that the people who make claims have to provide evidence)?

By Who Cares (not verified) on 11 Nov 2010 #permalink

Arcane distinctions like "nothing is ever proven" and "theory instead of law" just give exploitative types like the Koch brothers ammunition and wiggle room. We need to stop this. We need to say, "It is proven beyond a reasonable doubt," and "Law of Evolution" and "Law of Global Warming."

By puf_almighty (not verified) on 11 Nov 2010 #permalink

FB: "I hope you realize that comics do not constitute scientific arguments. Nor is your comic even relevant to what is being discussed..."

Here, let me spell it out for you:

"You are trying to model the behavior of global climate? Just model it as a linear system, and then add some stochastic noise to account for nonlinearities."

If you still don't get it I can't help you.

FB: Frankly, there is absolutely no evidence of high nonlinearity...

I see you now acknowledge that there IS non-linearity, that's a step forward at least.

Me: "there is no reliable way of translating forcing increase due to extra CO2 into global temperature rise."

FB: "This claim is unsupported by any physics or data."

Me: "The burden of proof is on those who claim it can be done."

FB: "What "burden of proof"? There is an enormous body of evidence linking CO2 to temperature changes through deep to shallow paleoclimate into historical times..."

I am not talking about evidence linking CO2 to temperature, I am talking about reliable way of translating forcing increase due to extra CO2 into global temperature rise. As you yourself stated in the last post "there is no "one estimate" of climate sensitivity" - which is exactly my point.

FB: You've missed the point. Millennial-scale slips in the glacial cycle are linked to millennial-scale processes (ice sheet dynamics) which are not present today.

No, you've missed it. Go back and reread the comments, the whole point was that there have been many periods when there was no correlation between CO2 levels and temperature or the correlation was negative. Why it was varying is completely irrelevant, as what I was showing was that the guy who claimed rising CO2 always equals rising global temperatures was wrong. If you were talking about something else then you got off topic.

Me: Straw man, my position is not that it is impossible to guesstimate sensitivity, but that there is no way to experimentally verify which guesstimate - and different people come up with very different ones - is actually correct now.

FB: As I said myself, there is no experimental way to "verify" what the equilibrium response to doubled CO2 is without actually doubling the CO2 concentration of the planet and waiting. Your argument appears to be that this therefore means that estimates have no "credibility", which is nonsense.

Yes, it means precisely that the credibility of such predictions is very low. There is only one source of credibility - experimental confirmation.

FB: As for different people coming up with different sensitivities, I already said that there is no "one estimate" of climate sensitivity. Because the data are coarse and sparse, there is a range of allowed sensitivities, and we may never narrow that range based on paleoclimate evidence alone. This, again, is different from claiming that the currently allowed range of sensitivities is not credible.

My point is that each particular estimate is not credible, the whole range may be credible but so what? That doesn't help us at all since it covers scenarios from very mild to extreme global warming.

FB: You apparently do not understand the scientific method yourself, because your representation of it sounds like something you'd hear from an 8th grade textbook, not a practicing scientist. In particular, you don't seem to think that observational sciences constitute credible science (which would include much of geology, astronomy, evolutionary biology, etc.).

Unfortunately you are right, many practicing scientists would do well to reread those 8th grade textbooks, that might remind them that, as long as there is no experimental verification, their pet theories are next to worthless and almost certainly wrong (from historical perspective only a tiny fraction of all proposed theories turns out to be right).

And an observation IS and experiment only not a controlled one.

FB: What matters to science is not controlled experiments per se, although those are great when available. What matters is theoretical frameworks that are consistent with the body of empirical evidence. You can't go back in time and re-run a glacial-interglacial cycle, but you can test which hypotheses about such cycles are compatible with the collection of geologic and biologic data available, as well as with modern observations and tested physical laws.

Wrong, the obvious counterexample is philosophy. Theoretical frameworks consistent with limited evidence are a dime a dozen, the absolutely critical part is being able to discriminate between them and tell the right ones from the wrong ones and that can only be done by experimental verification which usually requires controlled experiments.

Observational sciences like astronomy ARE much less credible then hard sciences where controlled experiments are cheap and easy. Evolutionary biology is not strictly an "observational science," evolution has been done in the lab, besides the fact that the number of subjects is so vast greatly helps, but of course those aspects of evolution which have been successfully reconstituted in the controlled laboratory experiments are more credible then those which are only theorized based on observational evidence.

'Is it incontrovertibly, scientifically proven that the increase in CO2 causes global warming?

No, no it isn't.'

is totally wrong. It has been proven over and over again that CO2 is a greenhouse gas in that it is a strong absorber of IR light. In fact, we have CO2 lasers based on the exact same physics that describe the CO2-forced greenhouse effect. All of this is based on quantum mechanics, is which arguably the most well-tested theory science has produced so far.

I think the OP's point can be summarized as "global climate change occurring due to CO2 emissions from human activity has not been proven beyond an unreasonable doubt."

Azkyroth,

'I think the OP's point can be summarized as "global climate change occurring due to CO2 emissions from human activity has not been proven beyond an unreasonable doubt."'

I think that should be the take home point, even in the event Ethan didn't mean that.

"You are trying to model the behavior of global climate? Just model it as a linear system, and then add some stochastic noise to account for nonlinearities."

Exactly. As far as global temperature is concerned, that is in fact an excellent approximation, unless you've crossed a major threshold like a thermohaline circulation collapse, which we are unlikely to experience in the next century or two. In the absence of such thresholds, even a linear energy balance model will do well.

I see you now acknowledge that there IS non-linearity, that's a step forward at least.

As I said before, linearity is an excellent approximation for the situation I'm discussing.

I am not talking about evidence linking CO2 to temperature, I am talking about reliable way of translating forcing increase due to extra CO2 into global temperature rise. As you yourself stated in the last post "there is no "one estimate" of climate sensitivity" - which is exactly my point.

Given a climate sensitivity, there is a reliable way of translating CO2 to temperature. Given a range of sensitivities, there is a reliable way of translating CO2 to a range of temperatures. This is science, after all, and calculations come with error bars on them.

Go back and reread the comments, the whole point was that there have been many periods when there was no correlation between CO2 levels and temperature or the correlation was negative.

I already explained this.

Why it was varying is completely irrelevant, as what I was showing was that the guy who claimed rising CO2 always equals rising global temperatures was wrong.

maxwell claimed no such thing. He claimed that rising CO2 enhances the greenhouse effect which causes warming. Whether other factors can counteract this warming is a separate question.

Yes, it means precisely that the credibility of such predictions is very low.

Nonsense. The fact that estimates have error bars on them doesn't mean that the estimates, with error bars are not credible. "Not credible" means that the error bars are wrong too.

There is only one source of credibility - experimental confirmation.

This is more nonsense, and completely at odds with all observational sciences. Experimental confirmation is certainly the best means to judge predictions, but that doesn't mean that credible predictions cannot be made.

My point is that each particular estimate is not credible, the whole range may be credible but so what? That doesn't help us at all since it covers scenarios from very mild to extreme global warming.

It covers about 2 to 4.5 C for 2xCO2, which is a wide range, but not so wide to be useless from either a scientific or a policy standpoint.

Unfortunately you are right, many practicing scientists would do well to reread those 8th grade textbooks, that might remind them that, as long as there is no experimental verification, their pet theories are next to worthless and almost certainly wrong

As another poster said, you have a cartoon picture of science. Empirical tests of hypotheses != controlled epxerimental verification.

And an observation IS and experiment only not a controlled one.

In that case, there is plenty "experimental" evidence for what I've said.

Wrong, the obvious counterexample is philosophy.

Philosophy is not a counterexample to the scientific validity of astronomy, geology, or any other field in which you cannot perform manipulative experiments. These fields, along with climate science, are supported by quite a bit of objective and quantitative data which is required to conform to physical laws.

Observational sciences like astronomy ARE much less credible then hard sciences where controlled experiments are cheap and easy.

They're not as cleanly supported by data, for sure, but that doesn't mean they're "not credible". That is, simply, ridiculous.

Evolutionary biology is not strictly an "observational science," evolution has been done in the lab,

Yes, and you can do radiative transfer in a lab, but this is beside the point.

Styx:

An XKCD cartoon is no substitute for the peer-reviewed literature. If you are going to link to anything to support your claims vis. global warming, link to the peer-reviewed literature. A webcomic, no matter how amusing or how analagous you feel it is (and XKCD certainly counts as amusing) just doesn't cut it.

Incidentally, the GIF you link to on Skeptical Science does not appear to support what you say it supports. Maybe you should find one of the Skeptical Science articles that the image appears in, which actually discusses the scientific literature on the topic, and review it.

Also, if experimental verification is so important, kindly link to the experimental verification supporting plate tectonics. We've done lab work re-creating earthquakes, right? Otherwise it's just an observational science with minimal credibility. Right?

On the other hand, the greenhouse effect (the atmosphere warming the Earth's surface) was experimentally verified by Tyndall. In the mid-nineteenth century.

Also, you may wish to consider that real-world measurements of, say, sea level rise, have thus far almost invariably been at the high end of past IPCC ranges of estimates. Uncertainty is not our friend in this matter: the error bars stretch in both directions.

Ethan, it might have been better to cut out the first section of the post and concentrate on the mechanism of absorption and re-emission of radiation. If more people understood the basic physics involved, they would realize that increasing concentrations of CO2 leading to higher atmosphere temperatures is a plausible assumption, not some outlandish, improbable speculation cooked up by a cabal of scientists.

By CherryBomb (not verified) on 11 Nov 2010 #permalink

FB: Exactly. As far as global temperature is concerned, that is in fact an excellent approximation...

No, this is your wishful thinking.

FB: Given a climate sensitivity, there is a reliable way of translating CO2 to temperature. Given a range of sensitivities, there is a reliable way of translating CO2 to a range of temperatures. This is science, after all, and calculations come with error bars on them.

Yes, only the range/error bars are so large as to make those calculations useless.

FB: maxwell claimed no such thing. He claimed that rising CO2 enhances the greenhouse effect which causes warming. Whether other factors can counteract this warming is a separate question.

The point is that seeing warming we cannot conclude to what extent it is caused by anthropogenic emissions of CO2 since there are plenty of other factors involved which can easily overcome CO2 contribution.

FB: Nonsense. The fact that estimates have error bars on them doesn't mean that the estimates, with error bars are not credible. "Not credible" means that the error bars are wrong too.

Nonsense. They are not credible because they are not experimentally verified.

FB: This is more nonsense, and completely at odds with all observational sciences. Experimental confirmation is certainly the best means to judge predictions, but that doesn't mean that credible predictions cannot be made.

And this is even more nonsense, as I said an observation IS an experiment.

FB: It covers about 2 to 4.5 C for 2xCO2, which is a wide range, but not so wide to be useless from either a scientific or a policy standpoint.

No, it may be as low as 1 C per Lindzen.

FB: As another poster said, you have a cartoon picture of science. Empirical tests of hypotheses != controlled epxerimental verification.

Get real, empirical tests of hypotheses = experimental verification.

FB: Philosophy is not a counterexample to the scientific validity of astronomy, geology...

Nice spin, philosophy is a counterexample to your naive notion that science is primary about theories.

Holy shit, Styx!

You're a fucking moron!

By Ema Nymton (not verified) on 11 Nov 2010 #permalink

Composer99: Incidentally, the GIF you link to on Skeptical Science does not appear to support what you say it supports.

Look again, there are plenty of periods where rising CO2 is accompanied by decreasing temperature.

Composer99: Also, if experimental verification is so important, kindly link to the experimental verification supporting plate tectonics. We've done lab work re-creating earthquakes, right? Otherwise it's just an observational science with minimal credibility. Right?

Once again observations are experiments! The problem with climate science is not that there is no experimental evidence for anything, but that there is not enough experimental evidence to support the claim that man-made emissions are the primary driver of recent warming. There is also not enough experimental support for climate models and therefore future climate predictions.

@"Ema Nymton"
Haha, love you too.

Styx: Shove it. If you have real arguments, write it up, submit it, get it published, then cite it. Otherwise, I don't see why we should bother reading your nonsense.

By Lotharloo (not verified) on 11 Nov 2010 #permalink

Can I suggest a simple way of resolving the dispute re: global warming.

Let's all do a prospective study. We know the average slope of warming during the 20th century. We have models predicting increased warming with increased CO2 levels. Barring a 2nd Great Depression, it is likely that CO2 levels will continue to rise.

If most of the global warming in the 20th century is due to a warmer sun then, fortunate for all of us, the solar cycle seems to be slowing down a lot recently and it is not likely to reverse and speed up significantly because, at the end of the 20th century, it was at one of its fastest cycles in recorded history.

So, here's how it works. If CO2 is the major cause of the 20th century's warming then we would expect for global surface temperatures to continue going up. So, beforehand, set a reasonable threshold between the expected and observed. When it crosses the threshold, well...we declare that anthropogenic CO2 is probably not the major cause of 20th century warming.

Of course, the solar cycle theory could be falsifiable using the same process. It is unreasonable to expect the solar cycle to go to 7 years. So, the sun isn't going to cause any more significant heating in the near future. But...if temperatures go up past some reasonable threshold, then we declare that anthropogenic CO2 is likely the major cause of 20th century warming.

How about it. If we start with 1999 I think that the argument would probably be resolved in 5 to 10 years. Since then, there has been about a 30% increase in anthropogenic CO2. If anthropogenic CO2 is the dominant cause of global warming then temperatures should have gone up about 0.3 C. They haven't gone up that amount so the deniers are ahead at this point in the game.

I would suggest setting a discrepancy of 0.5 C as the threshold. If 1/2 half of the 20th century's warming cannot be attributed to increased CO2 levels then the CO2 theory really ought to be called into question. If the deniers are correct, then, at the current rate, they will have their win in about 8 more years. If the CO2-ers are correct then there's no way that global surface temperatures can keep for warming when anthropogenic CO2 has gone up by about 50ppm.

It could be sort of fun to set up a public challenge like this. The challenge could be something that both sides might agree to since they both think that nature is on their side and that they will finally silence the other side and get the policy changes that they want.

If one side or the other doesn't want to go along with the public challenge...that in and of itself might be revealing. But the challenge should go on anyhow because we need resolution of this matter. If CO2 is a major driver of global warming then we need get on with the business of controlling CO2 emissions. Setting a criteria for action might get a breakthrough on the international level. If the sun is the major driver of global warming then science will experience an explicit collapse of a majorly promoted theory rather than such a major theory going quietly into the night.

How about it???

JohnHunt,

there is no need to do such a study. The only person here arguing about attribution is Styx. The post itself is about the greenhouse effect, an experimentally and observationally verified aspect of our reality on this planet. Given the physics of light absorption and emission, the very small variations in total solar output and decrease in temperature at higher layers of the atmosphere, we know human emitted CO2 is leading to an increased greenhouse effect.

That is all this post is about.

It's not about how more warming we'll have or how much of the past warming can be attributed to people with 100% confidence. We don't need to have 100% confidence in attribution to know that putting more absorbers into a sample (the atmosphere) will increase the sample's optical density. Because an excited molecule acts like a point source of radiation, emission of energy absorbed happens in all directions. Therefore some energy gets emitted back toward the surface of the earth. In response to this excess energy, the surface warms up. It will continue to warm until the amount of energy emitted equals the energy absorbed as per the second law of thermodynamics.

There is absolutely nothing controversial about any of that. In fact, it's the exact same basic physics that underlie the functioning of a blanket or a winter jacket. In response to increases in energy, an object will heat up until it is giving off the same amount of energy that it receives.

The fact that Styx so desperately wants to make this an argument about ice cores and attribution of past warming means that he/she can't understand the basic premise of this post. Whether or not humans are the cause of 100% of the warming we've measured so far does not in any way, shape or form negate the reality that adding CO2 to atmosphere increases the greenhouse effect which will increase the temperature of the surface of the earth, decrease the temperature of higher layers of the atmosphere and create a small, but possibly not measurable, difference in the top of the atmosphere fluxes of incoming and outgoing radiation.

End of story.

No, this is your wishful thinking.

Gee, somehow it's always everyone else's burden of proof, no matter the fact that you're the one who claimed the climate is so tremendously nonlinear. Well put up, or shut up.

The global temperature of the Earth, under the conditions I specified above, certainly is highly linear in climate models, and is also close linear in the data as far as anyone has ever been able to determine. Your evidence to the contrary is ... what, exactly?

Yes, only the range/error bars are so large as to make those calculations useless.

Knowing that climate sensitivity is probably around 3 C, and is probably not 1 C, or 5 C, is not at all "useless", from either a basic science standpoint or from a policy standpoint.

The point is that seeing warming we cannot conclude to what extent it is caused by anthropogenic emissions of CO2 since there are plenty of other factors involved which can easily overcome CO2 contribution.

The point is that we very well can make such attribution statements, because we are not ignorant of other factors involved. There are only a limited number of ways to make the surface temperature of the Earth increase, and we can rule most of them out directly. It's not solar variations, it's not heat from the ocean, it's not a large drop in aerosol loading, etc.

They are not credible because they are not experimentally verified.

This is an absurd requirement, that no prediction is "credible" until after it comes true. I have news for you: people make credible predictions about things all the time before they happen.

No, it may be as low as 1 C per Lindzen.

Lindzen is wrong, as you could tell by the followup rebuttals, or by basic common sense, as 1 C is fundamentally incompatible with anything ever happening in the paleoclimate record.

Get real, empirical tests of hypotheses = experimental verification.

Fine. In which case all my statements have "experimental verification", because they are supported by empirical data.

Nice spin, philosophy is a counterexample to your naive notion that science is primary about theories.

I didn't say science is primarily about theories. I said science is about constructing theoretical frameworks that are consistent with the body of empirical evidence. Quantitative comparison with empirical data is the whole point, and climate science has plenty of it, as do all other observational sciences.

I fail to see how regugitating the same old points and charts again is supposed to hold any sway. This physicist had access to all the same data or better and he still called it a hoax. Flamers, flame on if you honestly find this statement illogical.

It is disappointing that this long post never mentions that the radiative physics climate forcing is logarithmically, not linearly related to concentration. Approximately, doubling CO2 raises the temperature 1C. Doubling it again (now a factor of 4) only increases it one more degree C.

In order to get scary scenarios, one must assume strong positive feedback mechanisms - which are most certainly NOT well proven - just hypothesized.

Another important point is that we do NOT have a good record of past temperatures - not even for the last 50 years. The hoops that paleoclimatologists jump through in an attempt to reconstruct past average temperatures are full of uncertainties. Even modern records suffer from geographic sparseness, very poor site locations, and poor calibration. Nobody should believe that we actually know the average global temperature of, say, last week, much less 100 or 200 years ago.

Finally, let's assume that the alarming scenarios are not only true but well proven. Even in that case, the coercive measures being proposed by many (including scientists) simply will not work. We can damage our economy all we want, and the Chinese and Indians (and other large developing countries) will continue to greatly increase their CO2 emissions. The net result of anti-carbon policies in the west will be to damage our economies, hurt the life prospects of those least likely to afford it, while not helping the problem, if there is one, any significant amount. It is not science, it is modern puritanism.

SChaser,

'It is disappointing that this long post never mentions that the radiative physics climate forcing is logarithmically, not linearly related to concentration.'

Why is that 'disappointing'? That statement is especially confusing to me given the fact that your very next sentence sums up the post as a whole.

If we add more CO2 to the atmosphere, the temperature will go up.

So if you agree with the basic premise of the post, more atmospheric CO2 increases surface temperatures, why does it matter if the author mentioned the explicit mathematical form of the Beer-Lambert law?

If we add more CO2 to the atmosphere, the temperature will go up.

So if you agree with the basic premise of the post, more atmospheric CO2 increases surface temperatures, why does it matter if the author mentioned the explicit mathematical form of the Beer-Lambert law?

Because the implications of logarithmic vs linear effects are dramatic. Any post explaining the basic, zero-feedback greenhouse effect should explain that doubling the CO2 concentration comes nowhere close to doubling the forced temperature difference.

Furthermore, because of the hotly debated policy implications of this subject, it is especially important that laymen understand this fact.

The simple fact that adding CO2 results in increased temperature justifies no policy action at all. It is the magnitude and the uncertainty that would be important (except it's all irrelevant since the BRIC countries are greatly increasing their CO2 emissions and will simply not agree to any significant restrictions).

Q:How many global warming deniers does it take to change a light bulb?

A: 50. 25 to deny that it's dark, 15 to argue that the dark is better, and 10 to hang out waiting for the Rapture to take them from the dark room, before it gets too dark. (The light bulb never gets changed)

Humans are going to oxidize every bit of everything flammable they can get their hands on, until the species goes extinct. We're not going to "stop doing it" until we're all dead. Just face up to this fact, Ethan. There's nothing you can say or do to circumvent this certainty, so there's no point trying. Look at the flack you're receiving right here in the comments section of your blog. You're either preaching to the choir or serving as patsy to denialists. Why do you even bother?

By darwinsdog (not verified) on 11 Nov 2010 #permalink

SChaser,

It is disappointing that this long post never mentions that the radiative physics climate forcing is logarithmically, not linearly related to concentration.

Yes, that's an important point.

In order to get scary scenarios, one must assume strong positive feedback mechanisms

Another important point.

- which are most certainly NOT well proven - just hypothesized.

Hardly. Such feedbacks are supported by modeling and observations of individual feedback mechanisms (such as water vapor), model and observational estimates of the total feedback in result to historic forcings, and similar estimates for numerous climates of the past (thee Holocene, Last Glacial Maximum, the PETM, the Phanerozoic, etc.)

Another important point is that we do NOT have a good record of past temperatures - not even for the last 50 years.

I sort of agree if you're talking about the late Holocene, insofar as I think the error bars are bigger than usually advertised. I don't think the overall temperature estimates are that far off. And we're better off during the glacial-interglacial cycle, despite being further back in time, since the signal is much stronger.

I completely disagree about the last 50 years.

Even modern records suffer from geographic sparseness, very poor site locations, and poor calibration.

Geographic sparseness mostly only a problem in polar regions for the surface land stations, and parts of the Southern Ocean. Even then, it really isn't that much of a problem, and the supposedly "undersampled" estimates agree rather well with the satellites during periods of overlapping coverage (and much of the land area is far oversampled). Ditto for "poor site locations"; despite much ado about urban heat islands and such, these effects are rather minor, particularly after adjusting for obvious urbanization, and similar treatment of calibration. (One exception is the 1940s sea surface bucket corrections, which I don't think have been sorted out.)

Nobody should believe that we actually know the average global temperature of, say, last week, much less 100 or 200 years ago.

This is a pretty absurd claim.

Even in that case, the coercive measures being proposed by many (including scientists) simply will not work.

Maybe. But this is an assertion, not a fact. It's not as if developing nations disbelieve in climate change. They simply have no incentive to reduce emissions when the major emitters don't. Classic game theoretic scenario.

And "coercive" measures? Any fan of the free market should welcome corrections to market distortions caused by negative externalities. Economically it's "good" to price "bads".

Mentioning that the Chinese and Indians will never comply with reducing greenhouse gas emissions which would damage their own economies may also be extended to include the rest of the world excepting of course any of those countries whose "leaders" are slated to receive and personally control massive cash benefits for keeping their own people economically underdeveloped or disadvantaged.

The current global "cap and trade/tax" proposals are not designed to address any real problems, but rather to grab the wealth of those nations which actually have any serious monies to grab while supporting a "spread the wealth" ideology which necessarily bears massive overhead expenses while actually having been designed to further consolidate worldwide political power. A statement of "I'm carbon neutral" somehow fails to impress me when the individual making it is flying thousands of miles in his own personal jet and being driven in numerous SUVs and/or limos between just as many huge mansions, most of which happen to include hydrocarbon heated swimming pools. (Yes, such people exist and one even got a Nobel prize for expressing his personal serious concern for the ecology of this planet.)

That is why human kind will very likely fully succeed in burning this house down, but never fear, the perceived problem will sort itself out eventually. Whether anyone is here to see it or not is the real question.

By Lloyd Hargrove (not verified) on 11 Nov 2010 #permalink
- which are most certainly NOT well proven - just hypothesized.

Hardly. Such feedbacks are supported by modeling and observations of individual feedback mechanisms (such as water vapor), model and observational estimates of the total feedback in result to historic forcings, and similar estimates for numerous climates of the past (thee Holocene, Last Glacial Maximum, the PETM, the Phanerozoic, etc.)

"supported by" is a nice phrase - but models of a such a chaotic system as the atmosphere are very weak - even the most ardent alarmist scientists agree that the most significant mechanism - clouds - are not handled at all well. While some feedbacks are supported, others are guesses. As for historic forcings, it would help if the historic data were not so noisy as to be mostly meaningless.

Even modern records suffer from geographic sparseness, very poor site locations, and poor calibration.

Geographic sparseness mostly only a problem in polar regions for the surface land stations, and parts of the Southern Ocean. Even then, it really isn't that much of a problem, and the supposedly "undersampled" estimates agree rather well with the satellites during periods of overlapping coverage (and much of the land area is far oversampled). Ditto for "poor site locations"; despite much ado about urban heat islands and such, these effects are rather minor, particularly after adjusting for obvious urbanization, and similar treatment of calibration. (One exception is the 1940s sea surface bucket corrections, which I don't think have been sorted out.)

The satellite data is only very recent, and does *not* agree well with the surface data. Only with the new ocean heat measuring systems just put in place will we have a reasonably accurate measurement. After all, near-surface air temperature itself is a pretty poor proxy for global warming.

Even in that case, the coercive measures being proposed by many (including scientists) simply will not work.

Maybe. But this is an assertion, not a fact. It's not as if developing nations disbelieve in climate change. They simply have no incentive to reduce emissions when the major emitters don't. Classic game theoretic scenario.

Your own statements here strongly support the assertion. Yes, it is a game theoretic scenario - and the answer is: the developing nations have grossly inadequate incentive to reduce emissions for the sake of AGW, and thus, they obviously will not do so for that reason. Without dramatic and unforseeable improvements in alternative energy, the huge developing countries will simply not have the political ability to do anything. After all, if the wealthy United States, and the wealthy Europe can't change anything, it is certainly reasonable to project that poor but developing countries will have an even harder time.

And "coercive" measures? Any fan of the free market should welcome corrections to market distortions caused by negative externalities. Economically it's "good" to price "bads".

The problem with externalities arguments in this case is that they cannot be measured in any meaningful fashion, so assigning a value (or cost) is pretty arbitrary. Also, any coercive measure, even a "good" one, tends to be gamed by folks with enough money at stake to out-think the regulators. For example, in the case of cap and trade or a carbon tax, the result would obviously be an exporting of carbon intensive industry - we would be buying even more of our "green" goodies from countries which are not even close to green - except in their marketing brochures - and we would be doing so at the same time that our own people would be feeling negative economic effects which would probably make the programs unsustainable (notice how Kyoto didn't actually work in Europe - where there was, in theory, strong support for it).

Put another way... the economic system is also a non-linear dynamic system, and messing with it is as unpredictable as forecasting future climate.

The graph prepared by the Oak Ridge National Laboratory showing concentrations of CO2 and emissions of CO2 in parts per billion is misleading.

Itâs all very well telling us that total emissions of man made CO2 was close to Zero in 1750 but it fails to inform us how much in the way of emissions from other causes, such as volcanoes and decaying land and marine flora and fauns, should be added to the annual totals from 1750.

As the graph stands it looks like someone is trying to lie with statistics.

Let's just for the sake of argument assume that AGW is true, potentially serious and requires action.

There are two courses of action possible:
Mitigation; decarbonising the economy.
Adaptation; adopting measures to reduce the effects, such as mass migration.

Mitigation is eventually going to happen whether we like it or not. Fossil fuels are eventually going to run out because reserves are finite. There might be 50 years of petroleum left, perhaps 2-300 years of coal (but use of coal should be reduced, because it is polluting; coal contains trace amounts of uranium and thorium and because so much coal is burned, a coal-fired power plant releases more radioactivity than a nuclear reactor), ?natural gas.

We are going to have to decarbonise the economy, so why not now while we still have cheap energy supplies?

The best way is a carbon tax, provided the revenue raised doesn't just disappear into the black hole of general government revenue. Jim Hansen has suggested that it should go as a dividend to all the adults of the country. Another way would be to earmark it for the development of other energy sources.

"The graph prepared by the Oak Ridge National Laboratory showing concentrations of CO2 and emissions of CO2 in parts per billion is misleading" (comment #66).

No it isn't; until the industrial revolution, the CO2 cycle was in rough balance. CO2 released by decaying plants and respiration by animals and plants was balanced by the CO2 absorbed by photosynthesis. CO2 released from volcanos was balanced by the subduction of ocean floor sedimentary rock.

By waynerobinson4 (not verified) on 11 Nov 2010 #permalink

@Alan L: The Oak Ridge plots (and numerous others from various sources) is not at all misleading. We have a very good idea of how much CO2 we humans are putting into the atmosphere due to combustion of coal, gas and oil; if anything our estimates are lower than actual rates. Some folks like myself don't even bother to waste effort trying to include things like CO2 due to forest fires started by humans, we take the generous line that all forest fires are natural occurrences.

So, based on our conservative estimate of CO2 emissions from burning coal, oil and gas we know that the earth is soaking up roughly 50% of our CO2 emissions and only about 50% of emissions remains in the atmosphere. If we were to immediately stop burning any oil coal and gas it would be reasonable to expect the atmospheric CO2 to slowly decline due to the actions of nature. The CO2 variation through warm and cold periods over the past 50000 years shows that the natural variation in CO2 really isn't much at all. We humans are entirely to blame for the current increase in atmospheric CO2. You have to go back to a time when humans did not exist if you want an atmosphere with equal or higher amounts of CO2.

You can also discard the notion that volcanoes are suddenly putting out more CO2. Volcanic CO2 has a 13C/12C isotope ratio which is very different from coal and oil and gas deposits because it is mineral and not biological in origin. The change in the atmosphere's carbon isotope ratio is yet another irrefutable proof that humans, not nature, are responsible for the increasing CO2 in the atmosphere.

By MadScientist (not verified) on 11 Nov 2010 #permalink

Kindly do not make the false claim that I expressed a notion that volcanoes are âsuddenlyâ putting out more CO2. The contribution of volcanoes is as minor as it has ever been, a mere 1%.

The graph is misleading.

Absolutely nothing, other than mankindâs CO2 output caused by industry, appears to have been included. It is impossible to believe that Earthâs flora and fauna contributed absolutely Zero CO2 to Earthâs atmosphere in 1750.

@Styx:
Next time please be so honest to link the article instead of cherry picking a single graph (and caption) from it

The article itself is about the Milankovitch cycle(s). Which (upon further reading) are oscillations in the path of the earth around the sun and the resulting change in the amount of energy received. At the moment all you have done is confirmed the claim by Maxwell and Madscientist that the issue is more complex then the extremely simplistic assumption that increased CO2 equals increased temperatures because it excludes other factors that influence either.

By Who Cares (not verified) on 11 Nov 2010 #permalink

even the most ardent alarmist scientists agree that the most significant mechanism - clouds - are not handled at all well.

Clouds are certainly not the "most significant mechanism". The most significant single factor is the water vapour feedback (which is not the same as "clouds").

Here's a question for all those arguing that climate is tremendously chaotic and non-linear: if that's the case, how come we have predictable seasons?

It is impossible to believe that Earthâs flora and fauna contributed absolutely Zero CO2 to Earthâs atmosphere in 1750.

Prior to industrialisation, sinks and sources were in balance. If they natural sources exceeded the capacity of natural sinks prior to industrialisation, then (given the length of the Earth's history) we'd see all the free O2 converted to CO2 already.

SChaser,

Put another way... the economic system is also a non-linear dynamic system, and messing with it is as unpredictable as forecasting future climate.

Changing the underlying paradigm of our economic system is something I'm willing to experiment with. Changing the climate is an experiment I'd prefer not doing at all.

BTW, How's that economic system, that you seem so fond of, doin for ya?
http://i289.photobucket.com/albums/ll225/Fmagyar/ExponentialFunction2.j…

By Fred Magyar (not verified) on 12 Nov 2010 #permalink

SChaser,

'Because the implications of logarithmic vs linear effects are dramatic.'

Huh? What are the 'dramatic implications'?

It seems to me that the whole point of this post is to give the idea that adding more CO2 to the atmosphere increases the greenhouse effect which increases temperature by some amount. That one model would lead to some percent more warming to zeroth order over another I think is beyond the scope of any layman, who likely doesn't know the difference between a linear and logarithmic increase.

On top of that,

'The simple fact that adding CO2 results in increased temperature justifies no policy action at all.'

is a matter of your opinion. Science does not define what outcomes the members of a democracy should choose for the world. Science merely gives us as good an impression of the way the world works so that we are armed with information to make a democracy work.

Both sides of this political debate play this same tune. Science dictates x. I think that it's a poor way of making a political argument to say the least.

The average temperature of the Earth is 59F (i.e.13C).

The average temperature of the Earth has risen +.7F in last 30 years.

Temperatures rise causes ice melts (i.e. Antartic, Greenland, glaciers) to raise sea level.

Melt ing "Antarctic and Greenland ice sheets... sea-level rise of about 80 meters,.. melting of all other glaciers ... sea-level rise of only one-half meter."

So what do you think: 80 meters or 1/2 meter?

"A sea-level rise of 10 meters would flood about 25 percent of the U.S. population, with the major impact being mostly ... in the Gulf and East Coast States."

Are we ready for super-Katrina?

@Alan L: I already pointed out that you're wrong - it is not only CO2 put out by industry which is counted. Everything is counted, but nature not only doesn't create a measurable change in its balance, but in addition nature (currently) soaks up about 50% of industrial emissions as well. There is absolutely no disagreement between scientists over what is causing the increased CO2 in the atmosphere. The fact has been established via several independent methods including the carbon isotopes. If you do not understand that you need to read more rather than continuing to write that the plots are misleading and that somehow information is being left out. If it were nature which were responsible for any significant portion of the increasing CO2 we would know it, but there is absolutely no evidence whatsoever for that case.

By MadScientist (not verified) on 12 Nov 2010 #permalink

Huh? What are the 'dramatic implications'?

It seems to me that the whole point of this post is to give the idea that adding more CO2 to the atmosphere increases the greenhouse effect which increases temperature by some amount. That one model would lead to some percent more warming to zeroth order over another I think is beyond the scope of any layman, who likely doesn't know the difference between a linear and logarithmic increase.

If "the whole point" were as you say, the alarmist rhetoric in front and behind the radiative physics would not be present. But it is. So the whole point of this post is propaganda - to imply effects while leaving out critical information, such as the magnitude of first order CO2 warming, and the magnitude and dispute about climate sensitivity.

It is supremely dishonest. It claims to set the issue to rest by explaining the basic science, but it addresses areas far beyond the correct science that comprises but a part of the post, while leaving out the information necessary to make even the slightest assessment of impact of that basic science.

Furthermore, nobody serious disputes the basic radiative physics that adding CO2 increases lower surface temperatures. So a post explaining that fact is irrelevant, but a post wrapping that with less well established science is misleading.

I continue to wonder why no one points out the plain ABSURDITY of the notion of climate change being a hoax. It's conspiracy theorizing at its most ridiculous. The idea that countless scientists would be publishing countless peer- and public-reviewed lies, managing to keep none of those in on the hoax from spilling the beans, with the ultimate goal of...what, exactly, do the scientists gain from a carbon tax or shift to energy-efficient devices or ANY of the proposed solutions? The usual casual dismissal of, "they want more grant money," doesn't even work because they're not saying we need more study.

So it's patently obvious that it's not a hoax, but deniers (who, unlike the scientists, always have something to gain, or as Ethan notes, think they have something to gain, when they really have a lot to lose) probably understand that calling it a hoax, absurd as that is, is LESS absurd than claiming, with no scientific understanding to back them up, that all these scientists are just WRONG.

I agree with all the other points about cleaner air, averting an oil crisis before it happens, reducing money going to state sponsors of terror, making use of advanced technology (which often proves to be superior in more ways than being green), and the billion others, but it seems that anyone claiming a hoax really ought to try to explain that before any allegedly serious discussion of those issues or the science even begins.

"It is impossible to believe that Earthâs flora and fauna contributed absolutely Zero CO2 to Earthâs atmosphere in 1750."

It is not only impossible to believe, it simply isn't true. However, in total over a period of a year nature puts out pretty much just as much CO2 as it takes away. The result is a seasonal variation in CO2 which is much larger than the annual increase due to fossil fuel combustion, but over a year it adds up to a zero net change. That has been going on for tens of thousands of years through cold and warm periods and we know the CO2 remained almost constant when averaged over a mere few years throughout that long period. In addition to that, there is absolutely no evidence that nature is now putting out more CO2 than it is taking up. In fact the evidence is that nature is taking up even more CO2 - but humans are pumping it out so fast that nature can't compensate.

By MadScientist (not verified) on 12 Nov 2010 #permalink

I continue to wonder why no one points out the plain ABSURDITY of the notion of climate change being a hoax. It's conspiracy theorizing at its most ridiculous. The idea that countless scientists would be publishing countless peer- and public-reviewed lies, managing to keep none of those in on the hoax from spilling the beans, with the ultimate goal of...what, exactly, do the scientists gain from a carbon tax or shift to energy-efficient devices or ANY of the proposed solutions?

While the range of skeptical opinions vary, the primary idea of a "hoax" is not unrealistic *in some specific ways.*

Climategate showed that scientists, not surprisingly, act as human beings and are willing to intentionally distort data, hide problems, and even suppress skeptical articles from being published. That is the behavior of hoaxsters.

Most scientists in the fields are not perpetrating any sort of hoax. But *some are*. Some are intentionally distorting their findings when they make policy prescriptions (and note that policy prescriptions are not at all part of science).

Beyond that, there are strong financial incentives for scientists to find confirming results, and strong financial and professional incentives for them to not publish negative results. This is a recognized problem in most modern science, but is far worse in the climate field. This is not hoaxsterism, but it is a form of corruption of the process, which rightly leads skeptics to be, well, skeptical.

These incentives are so strong that one can find papers in almost every field of science that tie into "climate change" - as a handle to help get grants or peer approval. I did a Google Scholar search some time back and found such absurdities as 20 papers relating surfing to climate change! This huge number of papers mentioning climate change is cited (dishonestly) as a huge number of papers affirming climate change.

But getting back to hoaxes... it is not surprising that many are making incorrect claims of hoaxes. The enormous, economically damaging, and coercive policies advocated by some climate scientists, along with the grossly over-exaggerated certainty they claim, appropriately leads to serious skepticism. In some people, that skepticism leads them to assume hoaxes, rather than what is really going on: dysfunctional science, the most public spokesmen of which are making unfounded (or poorly grounded) grand claims; corrupt processes driven by money and politics; herd-think by scientists as a natural defense against criticism; the abuse of the results of this science by non-scientists who have their own reasons for wanting AGW to be as alarming as possible.

So it's patently obvious that it's not a hoax, but deniers (who, unlike the scientists, always have something to gain

This is a canard too often raised, and is so wrong as to be insulting. The skeptics (deniers is an offensive and incorrect term) do NOT always have something to gaim. In fact, most have nothing to gain. PhD Climatologists of my acquaintance are mostly skeptics, but what they "gain" is the scorn of their "peers" if the go public with their skepticism, so most are in the closet. On the other hand, alarmists have plenty to gain - grants, favorable publicity, peer approval - these are non-trivial.

I agree with all the other points about cleaner air, averting an oil crisis before it happens, reducing money going to state sponsors of terror, making use of advanced technology (which often proves to be superior in more ways than being green),

"cleaner air"? What are you talking about? If you mean removing CO2, please apologize to the plants which LOVE increased CO2. Otherwise, what the heck are you talking about?

"advanced technology" so far has proven to be not even close to being economically competitive with burning hydrocarbons, for many good economic and engineering reasons. If you can get the Indians and Chinese to switch to those technologies, then you have an argument that we should. Until then, its mostly a huge waste of money.

That isn't to say that we shouldn't continue to research these technologies - but we should recognize that the current trends are not economically favorable. A green job is nothing but a trade of a couple of non-green jobs for a green job - hardly a good deal.

As for oil issues - yes, some day we have to deal with that. However, right now the easiest way to do is just to free up our own hydrocarbon resources - they are currently being held hostage by environmentalists and NIMBY's. The US and Canada have as much hydrocarbon reserve as the entire rest of the world - we just aren't allowed to use much of it (coal and oil, primarily).

SChaser @ 79:

Climategate showed that scientists, not surprisingly, act as human beings and yet are not willing to intentionally distort data, hide problems, and even suppress skeptical articles from being published.

Edited for accuracy.

SChaser: "Scientists even... suppress skeptical articles from being published"

No they don't. They TALK about this, when they're very angry--in that particular case over a specific journal that in 2003 behaved so unprofessionally that half its editorial staff resigned. They also talk about punching their critics in the nose. No such punches and no such article suppression ever occurred. Post proof or retract.

"ClimateGate" = fractal-caliber infinite repetitions of goalpost-shifting, and all devoted to nothing more than thoughtcrime.

SCHaser:

Deniers, is, unfortunately, not too strong a term to apply when discussing the like of Anthony Watts, Steven Goddard (apologies if I mis-spelled his first name), and Christopher Monckton.

Their behaviour on the subject of climate science is indistinguishable from the behaviour of the likes of Gary Null or Dana Ullman (quacks of the highest order both) discussing scientific medicine, or creationists attempting to take on evolution.

They are, in fact, denialists.

Also, the CO2 as plant food argument ignores all the other limiting factors on plant growth (water & other nutrients in particular), none of which are increasing in step with CO2 and whose availability in many regions will decrease due to the shifts caused by climate change from CO2 increase.

Certainly, plants have not seemed to have any problem growing in the pre-industrial era, with its cap of approximately 280 ppm CO2.

SChaser,

'Furthermore, nobody serious disputes the basic radiative physics that adding CO2 increases lower surface temperatures.'

The presumption behind this statement is that this post is meant as an addition to the already overblown internet-based argument concerning the specific science of climate change/global warming. I think such a presumption is flawed in that, in my opinion, this post seems to be aimed at an audience unfamiliar with the basic science behind the idea of global warming to any order.

I agree that there are a few statements which Ethan could have pointed out to his audience that represent his own personal opinion. I specifically called out his claims concerning the comparison of the earth's greenhouse effect to that of Venus because I don't think it's sound based on known principles of weather patterns on the respective planets (comment #4). However, those statements do not undermined the validity of the scientific claims concerning the basic science behind climate change/global warming. I think his statement,

'If we don't stop doing it, we will undoubtedly raise the Earth's temperature, and it becomes a question only of "by how much?"'

is most important.

Instead of focusing on the difference you may have with him concerning your political stance, why don't try to find some common ground from which a meaningful debate can begin without resorting to the use of names like 'alarmist' or 'denier'? I mean, irrespective of the highly uncertain nature of the science of climate forecasting, if the people want to deal with climate change, why shouldn't we?

SChaser said (comment 79)and I quote;
"("cleaner air")? What are you talking about? If you mean removing CO2, please apologize to the plants which LOVE increased CO2. Otherwise, what the heck are you talking about?"

OK, now you are really grasping at straws, you are on the ropes here *SChaser* and are well out of your league. You know very well that there is a multitude of other pollutants that are emitted from burning (fossil fuels).

SChaser, you are a troll, a shill, dishonest and worst of all willingly blind to that which causes damage to the ecology of our planet, and it is I that have no more time to waste with your lower IQ, good day.

How about going back to science:

Editor's Summary
28 January 2010
CO2 feedback recalculated

Letter
Nature 463, 527-530 (28 January 2010) | doi:10.1038/nature08769; Received 24 July 2009; Accepted 12 December 2009
Ensemble reconstruction constraints on the global carbon cycle sensitivity to climate
David C. Frank1,2, Jan Esper3, Christoph C. Raible2,4, Ulf Büntgen1, Valerie Trouet1, Benjamin Stocker2,4 & Fortunat Joos2,4
1.Swiss Federal Research Institute WSL, Zürcherstrasse 111, CH-8903 Birmensdorf, Switzerland
2.Oeschger Centre for Climate Change Research, University of Bern, Zähringerstrasse 25, CH-3012 Bern, Switzerland
3.Department of Geography, Johannes Gutenberg University, Becherweg 21, 55099 Mainz, Germany
4.Climate and Environmental Physics, Physics Institute, University of Bern, Sidlerstrasse 5, CH-3012 Bern, Switzerland
Correspondence to: David C. Frank1,2 Correspondence and requests for materials should be addressed to D.C.F. (Email: david.frank@wsl.ch).

The processes controlling the carbon flux and carbon storage of the atmosphere, ocean and terrestrial biosphere are temperature sensitive1, 2, 3, 4 and are likely to provide a positive feedback leading to amplified anthropogenic warming3. Owing to this feedback, at timescales ranging from interannual to the 20â100-kyr cycles of Earth's orbital variations1, 5, 6, 7, warming of the climate system causes a net release of CO2 into the atmosphere; this in turn amplifies warming. But the magnitude of the climate sensitivity of the global carbon cycle (termed γ), and thus of its positive feedback strength, is under debate, giving rise to large uncertainties in global warming projections8, 9. Here we quantify the median γ as 7.7âp.p.m.v. CO2 per °C warming, with a likely range of 1.7â21.4âp.p.m.v. CO2 per °C. Sensitivity experiments exclude significant influence of pre-industrial land-use change on these estimates. Our results, based on the coupling of a probabilistic approach with an ensemble of proxy-based temperature reconstructions and pre-industrial CO2 data from three ice cores, provide robust constraints for γ on the policy-relevant multi-decadal to centennial timescales. By using an ensemble of >200,000 members, quantification of γ is not only improved, but also likelihoods can be assigned, thereby providing a benchmark for future model simulations. Although uncertainties do not at present allow exclusion of γ calculated from any of ten coupled carbonâclimate models, we find that γ is about twice as likely to fall in the lowermost than in the uppermost quartile of their range. Our results are incompatibly lower (Pâ<â0.05) than recent pre-industrial empirical estimates of ~40âp.p.m.v. CO2 per °C (refs 6, 7), and correspondingly suggest ~80% less potential amplification of ongoing global warming.

Summary: 40/8=5. So: the value used before for this parameter is 5 times the newer value. What does this do to global warming studies and conclusions? âand correspondingly suggest ~80% less potential amplification of ongoing global warmingâ.

Published Online January 28, 2010Science DOI: 10.1126/science.1182488Science Express Index
Research Articles
Contributions of Stratospheric Water Vapor to Decadal Changes in the Rate of Global Warming

Susan Solomon,1 Karen Rosenlof,1 Robert Portmann,1 John Daniel,1 Sean Davis,1,2 Todd Sanford,1,2 Gian-Kasper Plattner3

Stratospheric water vapor concentrations decreased by about 10% after the year 2000. Here, we show that this acted to slow the rate of increase in global surface temperature over 2000 to 2009 by about 25% compared to that which would have occurred due only to carbon dioxide and other greenhouse gases. More limited data suggest that stratospheric water vapor probably increased between 1980 and 2000, which would have enhanced the decadal rate of surface warming during the 1990s by about 30% compared to estimates neglecting this change. These findings show that stratospheric water vapor represents an important driver of decadal global surface climate change.

1 NOAA Earth System Research Laboratory, Chemical Sciences Division, Boulder, CO, USA.
2 Cooperative Institute for Research in Environmental Sciences, University of Colorado, Boulder, Colorado, USA.
3 Climate and Environmental Physics, Physics Institute, University of Bern, Sidlerstrasse 5, 3012 Bern, Switzerland.

SChaser, you are a troll, a shill, dishonest and worst of all willingly blind to that which causes damage to the ecology of our planet, and it is I that have no more time to waste with your lower IQ, good day.

Posted by: LMhaser

And you call ME a troll?

Instead of focusing on the difference you may have with him concerning your political stance, why don't try to find some common ground from which a meaningful debate can begin without resorting to the use of names like 'alarmist' or 'denier'?

The term "denier" was applied to people holding my position, and it is at least as accurate to reply with the term "alarmist"

There are two fundamental problems with this article, which purports to inform:

1) It claims to provide information about the science, but wraps the well established radiative balance science in much less certain statements and policy prescriptions - while providing the same gloss of accuracy to all.

2) In the radiative balance section, it leaves out all quantitative information. Hence the uninformed reader is left with the correct but useless (without quantification) idea that adding carbon warms the planet.

I mean, irrespective of the highly uncertain nature of the science of climate forecasting, if the people want to deal with climate change, why shouldn't we?

Go ahead. Just don't pass laws based on it that destroy my economy. By all means don a climate hair shirt, live a low emissions lifestyle, and feel good about the whole thing. I have no objection to that. Even take a bit of my tax money and use it to research alternative energy sources (and continue researching the climate system).

But don't institute cap and trade, or carbons emissions limits, or other such economically destructive folly.

And finally, recognize the following:

1) Unless the BRIC countries make enormous and probably politically impossible sacrifices, nothing we do will make any difference.

2) The economic systems is also poorly understood, but the effects of tinkering with it can include mass starvation and warfare. These and related risks are almost always totally ignored by those who invoke some form of the precautionary principle about CO2 remediation.

3) The science of "climate change" is not settled, or even close to it. In fact, the embarrassing lack of warming in the early 21st century led the alarmists (I use the term advisedly here and in a limited sense) to change their favored terminology from "global warming" to "climate change". While most climate scientists are honest and careful people, they have not cracked some of the most important problems, such as the fact that the magnitude and even the sign of cloud feedback has not been established an my not be determinable for a long time.

4) Science is not about consensus, exception in a sociological sense. It is about testing hypothesis with experiments. Computer models of climate do not provide strong experimental evidence, for many technical reasons, and the use of them instead of true physical experiments amounts to more of a religious than scientific undertaking.

embarrassing lack of warming in the early 21st century led the alarmists to change their favored terminology from "global warming" to "climate change".

What ignorant balderdash! "Global warming" and "climate change" have been used more or less interchangeably for at least 20 years, since it was well-known from the start that an overall temperature increase could cause localized cooling events. You might as well try to cast sinister aspersions on the people who said "fusion bomb" instead of "hydrogen bomb."

Science is not about consensus, exception in a sociological sense. It is about testing hypothesis with experiments.

Science is all about consensus, as any real scientist could have told you if you tried to educate yourself about this topic. Ask any geneticist how many chromosomes humans have. Ask any astrophysicist whether black holes have strong gravity or weak gravity. Ask any MRI tech whether the heart is located in your chest or in your elbow. Ask any atmospheric chemist whether CO2 is a greenhouse gas.

Stop trying to bluff about environmental issues. You can get away with that on Free Republic, but not on ScienceBlogs.

"It is impossible to believe that Earthâs flora and fauna contributed absolutely Zero CO2 to Earthâs atmosphere in 1750."

Whoever wrote this is an ignorant imbecile.

The carbon cycle of the biosphere does not usually tend to raise atmospheric CO2 levels. Plants and other photosynthesizers respire CO2 from the atmosphere and fix it, mainly as carbohydrates, proteins, and lipids. Animals and many unicellular organisms eat carbohydrates, proteins, and lipids, and breath out CO2, which is respired by the photosynthesizers. It is a cycle that is essentially in balance.

However, a large amount of carbon is sequestered in deposits of so-called "fossil fuels" such as petroleum and coal. When humans oxidize these carbon sources at a high rate per unit time, a greater amount of carbon than can be balanced solely by the carbon cycle of the biosphere is potentially added to the atmosphere, per unit time.

These are basic facts. You may deny that we can accurately measure how much net carbon is added, you may deny that atmospheric carbon concentration can impact on global climate, you may deny that we can measure the impact of atmospheric carbon on global climate, you may deny that changes in global climate can have any net negative impact on humans, you may agree that it can have net negative impact but deny that we can predict what the net negative impact may be, you may even agree that we can measure the carbon, measure how it affects climate, predict negative impact, but still rabidly deny that anyone alive should make any personal sacrifice. All of these contradictory forms of denial are routinely issued, frequently by the same people within a short period of time.

At the end of the day, it is the last attitude that truly reflects the view of denialists. As denialist Richard D. sarcatically says -

Oh how horrible to be cursed by history after we are dead and past caring!

He just doesn't care, and that is really what they are all trying to say. Some of them are a bit more inhibited about expressing that view openly.

However, the carbon cycle is just neutral scientific fact.

There is no such thing as a "global warming skeptic".

A skeptic is one who attempts to take a clear and objective view of reality. Outright denial of the idea that large scale oxidizing of sequestered carbon sources over a relatively short period of time might have a measurable impact on atmospheric CO2 concentration, and thus on global climate (yes, other things impact on climate too), is not a "skeptical" position.

One can argue against taking action that would reduce net carbon emissions and still be a skeptic. Being a skeptic implies certain ethics with respect to honest discourse, but beyond that, skepticism refers to a disciplined way of observing reality - not to a means of deciding what is "good" and what is "evil".

Why not be honest?

Go ahead. Just don't pass laws based on it that destroy my economy. By all means don a climate hair shirt, live a low emissions lifestyle, and feel good about the whole thing. I have no objection to that. Even take a bit of my tax money and use it to research alternative energy sources (and continue researching the climate system).
But don't institute cap and trade, or carbons emissions limits, or other such economically destructive folly.

Yes, admit it. You don't give a damn, and would not voluntarily sacrifice one iota - even if the sacrifice were purely psychological. You would no more make a mild sacrifice for the sake of potentially mitigating future negative climate change, than you would voluntarily slow your SUV to avoid crushing some scabby mongrel dog.

Most of the time, most humans seem to care about their genetic descendants, but extreme environments - excessively cosseted ones, perhaps - seem to change that.

When one looks at the fascinating history of the mid-to-late Roman ruling classes, for example, or some Medieval or Ottoman royal or aristocratic families, one often sees a level of decadence (my subjective term) which causes a breakdown of such concerns. In such extreme environments, learned narcissistic urges seem to trump any concern for others, and we see many examples of decadent violence committed against close genetic relatives. Interestingly, these actions often prove self-destructive, too, in the long run. But the actors seem to have been obsessed with short term increases in luxury and prestige.

Go ahead and admit it more directly than you already have. Relieve the cognitive dissonance and revel in that feeling of relief that comes from speaking frankly. You. Just. Don't. Give. A. Damn.

And that, rather than convoluted webs of illogical denialism, is your true position.

embarrassing lack of warming in the early 21st century led the alarmists to change their favored terminology from "global warming" to "climate change".

What ignorant balderdash! "Global warming" and "climate change" have been used more or less interchangeably for at least 20 years, since it was well-known from the start that an overall temperature increase could cause localized cooling events. You might as well try to cast sinister aspersions on the people who said "fusion bomb" instead of "hydrogen bomb."

Apparently you have not been paying attention to the public debate. The popular term, pushed by the scientists advocating strong policies, was "Global Warming" or "Anthropogenic Global Warming," until the earth (perhaps temporarily) stopped warming. Then the popular term became "Climate Change." Nice try, though.

Science is not about consensus, exception in a sociological sense. It is about testing hypothesis with experiments.

Science is all about consensus, as any real scientist could have told you if you tried to educate yourself about this topic. Ask any geneticist how many chromosomes humans have. Ask any astrophysicist whether black holes have strong gravity or weak gravity. Ask any MRI tech whether the heart is located in your chest or in your elbow. Ask any atmospheric chemist whether CO2 is a greenhouse gas.

You are confusing the sociology of science with the methods. Of course scientists use the results of other scientists, and agree on it once it is well established. The high feedback predictions of global warming are not well established.

Ask any serious scientist if they take a vote to determine reality.

Stop trying to bluff about environmental issues. You can get away with that on Free Republic, but not on ScienceBlogs.

Yeah, obviously anything disputing the "consensus" must be bluffing. Obviously anyone disputing the wisdom handed down by the prophet Hansen is an ignoramus.

Your argument by authority is a sign of the weakness of your position.

A skeptic is one who attempts to take a clear and objective view of reality. Outright denial of the idea that large scale oxidizing of sequestered carbon sources over a relatively short period of time might have a measurable impact on atmospheric CO2 concentration, and thus on global climate (yes, other things impact on climate too), is not a "skeptical" position.

One can argue against taking action that would reduce net carbon emissions and still be a skeptic. Being a skeptic implies certain ethics with respect to honest discourse, but beyond that, skepticism refers to a disciplined way of observing reality - not to a means of deciding what is "good" and what is "evil".

All of that is true.

Why not be honest?

And this means, what?

Go ahead. Just don't pass laws based on it that destroy my economy. By all means don a climate hair shirt, live a low emissions lifestyle, and feel good about the whole thing. I have no objection to that. Even take a bit of my tax money and use it to research alternative energy sources (and continue researching the climate system).
But don't institute cap and trade, or carbons emissions limits, or other such economically destructive folly.

Yes, admit it. You don't give a damn, and would not voluntarily sacrifice one iota - even if the sacrifice were purely psychological. You would no more make a mild sacrifice for the sake of potentially mitigating future negative climate change, than you would voluntarily slow your SUV to avoid crushing some scabby mongrel dog.

(other insults deleted)

Why don't you just admit that you are in favor of punitive policies because it makes you feel good? Why don't you just admit that you act as if you are scared to death of climate change, but your emotional investment in modern puritanism has kept you from seeing the serious problems with the policy prescriptions, including:

1) That the policies may prove terribly harmful, probably causing massive loss of life as a result of the trickle-down effect of a damaged global economy on those living on the margin. That these changes might lead to wars. That they might lead to a long term discrediting of science, which would be harmful to all? You seem to assert with great certainty that hthis is a "mild sacrifice," when many serious experts in economics (are you an economist) strongly disagree.

2) That the policies are not going to make any differece in the CO2 composition of the atmosphere, because the BRIC countries not only do not want to suffer from these policies, but are politically unable to do so without internal strife and the fall of governments.

Go ahead and admit it more directly than you already have. Relieve the cognitive dissonance and revel in that feeling of relief that comes from speaking frankly. You. Just. Don't. Give. A. Damn.

And that, rather than convoluted webs of illogical denialism, is your true position.

And your true position is obviously that mankind is a blight, and we should remove it by cutting our carbon emissions back to the stone age.

No, I don't believe that is your position, but is as good an extrapolation of your argument as your contemptuous and insulting extrapolation of mine.

Certainly rudeness and lack of critical thinking skills are not confined to the unwashed masses, as is clear from your posts.

I would suggest that your views are a result of the natural human need for religious activity, and your religion is that of scientism.

SChaser:

"...your religion is that of scientism."

OOpsie. There goes whatever credibility you may have had.

By Gingerbaker (not verified) on 14 Nov 2010 #permalink

There is absolutely nothing controversial about any of that. In fact, it's the exact same basic physics that underlie the functioning of a blanket or a winter jacket. In response to increases in energy, an object will heat up until it is giving off the same amount of energy that it receives.

There is no such thing as a "global warming skeptic".

Actually, I'm inclined to disagree. There are plenty of people who have taken a clear, objective view of global warming. They're just not the same people who claim to be sceptics.

SChaser: Apparently you have not been paying attention to the public debate

As I pointed out, I've been part of the public debate on the pro-science side for over 20 years. So when I say "stop bluffing", part of what I mean are your Johnny-Come-Lately attempts to tell me who "really" said what and when. Next you'll probably say we were all worried about global cooling in the '70s, because that's what the radio told you about our history too.

And yes, anyone who disputes the mechanism of the greenhouse effect and the anthropogenic inputs thereto is bluffing--at best.

"and the most abundant one on Earth is carbon dioxide, or CO2".

No, the most abundant is H2O - water vapor.
CO2 concentration is something like 0.038%, water vapor concentration is typically 1%-4% at surface.

There is no doubt that CO2 emissions make world warmer. There are doubts that they are making it *dangerously* warm.

My opinion is that in last few millions of years plants become very efficient in absorbing CO2, lowered its concentration almost to 0%, and in this way made world dangerously *cold*. Ice Ages are the evidence.

Plants will use up our CO2 emissions soon enough (in a blink of an eye in geologic terms), considering that we can't produce them forever: there are limited oil resources on this planet.

If the cat is dead; it is someone else's cat.

If the cat is both dead and alive; it is Schroedinger's cat.

Global warming is the Schroedinger's cat of quantum politics.

A global warming disaster is and will always be someone else's cat.

By AngelGabriel (not verified) on 15 Nov 2010 #permalink

Hide the decline! Hide the decline! Hide the decline! Hide the decline! Hide the decline! Hide the decline! Hide the decline! Hide the decline! Hide the decline! Hide the decline! Hide the decline! Hide the decline! Hide the decline! Hide the decline! Hide the decline! Hide the decline! Hide the decline! Hide the decline! Hide the decline! Hide the decline! Hide the decline! Hide the decline! Hide the decline! Hide the decline! Hide the decline! Hide the decline!

By Michael Mann (not verified) on 15 Nov 2010 #permalink

"...your religion is that of scientism."

Opsie. There goes whatever credibility you may have had.

Oopsie - perhaps you don't know the difference between "scientism" and science or the practice thereof.

As I pointed out, I've been part of the public debate on the pro-science side for over 20 years. So when I say "stop bluffing", part of what I mean are your Johnny-Come-Lately attempts to tell me who "really" said what and when. Next you'll probably say we were all worried about global cooling in the '70s, because that's what the radio told you about our history too.

I have been closely following the debate for a long time, and the public debate has definitely shifted from "Global Warming" to "Climate Change." It sounds like you are the denier.

And you just can't avoid the insults, can you, with asinine statements like " because that's what the radio told you about our history too" as if you know where I get my knowledge. Hint... I have lived the history. I don't need the radio to tell me what it was. I don't even listen to the radio.

And yes, anyone who disputes the mechanism of the greenhouse effect and the anthropogenic inputs thereto is bluffing--at best.

Strawman alert! Who is disputing that here? Certainly I have not, nor do serious skeptics (you know, those who methodically question the issues, rather than the strawmen "deniers" you keep invoking).

As I started out with in this whole string, the problem is magnitude, not the simple, well understood, widely agreed upon radiative physics of CO2.

Another problem is the idea, often pushed implicitly or explicitly by those favoring drastic policy changes, that the simple radiative physics proves the alarming predictions. This effect is achieved by not explaining to the layman the necessity of strong positive feedback in order to achieve catastrophic changes, and by leaving out the important logarithmic characteristic of the radiative balance.

Anyone who says "the science is settled" is either a liar or doesn't understand the subject.

SChaser -

Your entire post seems to consist of false dichotomies.

Why don't you just admit that you are in favor of punitive policies because it makes you feel good?

Because neither of these things would be true. I do not favor punitive policies, and punitive policies would not make me feel good. Of course, "punitive" is a subjective term.

False dichotomy - Either punitive policies or no policies at all.

Why don't you just admit that you act as if you are scared to death of climate change, but your emotional investment in modern puritanism has kept you from seeing the serious problems with the policy prescriptions, including:
1) That the policies may prove terribly harmful, probably causing massive loss of life as a result of the trickle-down effect of a damaged global economy on those living on the margin. That these changes might lead to wars. That they might lead to a long term discrediting of science, which would be harmful to all? You seem to assert with great certainty that hthis is a "mild sacrifice," when many serious experts in economics (are you an economist) strongly disagree.
2) That the policies are not going to make any differece in the CO2 composition of the atmosphere, because the BRIC countries not only do not want to suffer from these policies, but are politically unable to do so without internal strife and the fall of governments.

False dichotomy - either take no action or take action that has dramatically bad impact.

False dichotomy - India will either develop along a "cheap foreign oil" model or not develop at all. I can't speak much for Brazil and China, but I have many business associates and friends from India or of South Asian descent. Almost all of them believe that India should develop in a way that includes energy self-sufficiency and environmental sustainability.

And your true position is obviously that mankind is a blight, and we should remove it by cutting our carbon emissions back to the stone age.

No, it isn't.

No, I don't believe that is your position, but is as good an extrapolation of your argument as your contemptuous and insulting extrapolation of mine.

Well, glad you didn't really believe that. I don't agree that the position is analogous. I extrapolated that you don't care. Your post here supports my earlier extrapolation.

Certainly rudeness and lack of critical thinking skills are not confined to the unwashed masses, as is clear from your posts.

My critical thinking skills are excellent, whereas yours cannot be evaluated. One who makes illogical arguments and denies reality may secretly possess strong critical reasoning skills. As for rudeness you are at least my equal.

It's not my habit to refer to other people as "unwashed masses" (*of course this description may be literally true of people who live in areas where water is a scarce resource*), and says a great deal about your biases that you do so.

I would suggest that your views are a result of the natural human need for religious activity, and your religion is that of scientism.

You would suggest quite wrong. Scientism is not my perspective. Another attempt at a false dichotomy - deny science or be guilty of "scientism". http://en.wikipedia.org/wiki/Scientism

However, this is irrelevant. Scientism refers to a philosophical stance that subjectively reveres science to what some may perceive as an exaggerated degree.

The issue here is that you are denying science (and logic) out of loyalty to an ideology and false perception of what your self interest is. "Scientism" has nothing to do with it.

False dichotomy - Either punitive policies or no policies at all.

The policies recommended to achieve a significant impact (assuming, magically, that the BRIC countries will go along) are terribly punitive. Too many proponents of reducing CO2 emissions are dishonest about this. BTW, cap and trade is punitive - if it is to have an effect - because it only works by increasing the price of carbon emissions to a level where it is painful - sounds punitive to me.

If I thought reducing CO2 emissions to a level to make a difference (if one believes alarmist predictions) would be anything other than a serious drag on the economy, I would be more willing to go along.

The issue here is that you are denying science (and logic) out of loyalty to an ideology and false perception of what your self interest is. "Scientism" has nothing to do with it.

I see.. what would that ideology be? Oh, and please explain how you understand my self interest and I don't.

What arrogance. No wonder there is so much push-back against the alarmists.

But in all seriousness, beyond the verbal sparring, you have yet to address the issue of the BRIC countries. Do you advocate that America adopts expensive CO2 mitigation activities if these countries do not? Please be specific.

the public debate has definitely shifted from "Global Warming" to "Climate Change"

The "global warming" vs. "climate change" usage discussion is very tedious, but has an interesting history.

First, there are all sorts of climate changes, global warming being the one discussed here. The phrase "global warming" was coined by geochemist Wally Broecker in an 1975 article in Science (I think). For scientists studying climate change "global warming" is shorthand for "climate change, specifically the rising of Earth's surface air temperature when averaged over the entire globe, resulting from humans burning fossil fuels and putting lots of extra CO2 in the atmosphere".

To a climate scientist, implicit in the phrase "global warming" is that the warming will not be equal over the globe. Some places will get a lot warmer, others only moderately so, while a few places might actually cool. Some places will warm more in winter than summer and others not. There might also be a difference in daytime warming vs. nighttime warming. It is also fundamental knowledge to a climate scientist that the spatial variability in the heat balance of the earth-atmosphere-ocean system will result in changing atmospheric and oceanic circulation patterns, thus also changing global precipitation patterns.

As I said all that knowledge that hidden away in the two-word phrase is understood by people who study the climate. Most of that information is lost when a non-expert hears the phrase "global warming". For better or worse, over the years the phrase has been generally come to mean "bad things will happen to the climate in the future if we don't curtail our use of fossil fuels."

Politically, the people who think we should do something to prevent "bad things from happening in the future" have used the negative connotation of "global warming" to advance their political position.

So, how do people who don't think burning fossil fuels will cause bad things to happen to the climate, or have a vested interest in not curtailing fossil fuel, or are opposed to government mandating changes to individual energy choices, wrestle the terms of the debate from "global warming = bad" to something else?

Enter Frank Luntz, Republican pollster and public relations genius. Early on the Bush administration Luntz did some polling and figured out that "global warming" has strong negative connotations, but that "climate change" does not (it is perceived as a softer, less scary, phrase). Based on his research Luntz advised the Bush administration (google Luntz global warming memo) to always use "climate change" in place of "global warming".

So, SChaser, the idea to use "climate change" instead of "global warming" is one that comes from conservatives who do not want to take action to address the issue of increasing atmospheric greenhouse gas concentrations.

What's fascinating to me is that so many conservatives think that a) rephrasing "climate change" as "climate change" was some sort of sneaky thing liberals did, and b) the rephrasing somehow changes... something. Well, I've never quite been able to figure out what the point of this line of argument is. I've probably seen this same point made hundreds of times by commenters on climate blogs. Each time the commenter seems to think they've had a brilliant insight or they've just cleverly trumped the argument of whoever they are arguing with. In reality though, it doesn't change the physics of the greenhouse effect, the fact that humans are putting more CO2 into the atmosphere faster than it can be removed and that the resulting increase in CO2 is warming the climate and acidifying the oceans.

SChaser:" "...your religion is that of scientism."

Opsie. There goes whatever credibility you may have had.

Oopsie - perhaps you don't know the difference between "scientism" and science or the practice thereof. "

I know the difference, all right, which is why I called you out on your denialist-diagnostic, pathetically foolish, off-topic, and completely gratuitous non sequitur.

Seriously - "your religion is that of scientism"? Why not just tattoo "I am a right-wing moron" on your forehead and be done with it?

By Gingerbaker (not verified) on 15 Nov 2010 #permalink

Although your comment contains insults (inaccurate ones, too), I'll try to keep further insults out of mine for clarity. I will answer your questions, and I have some questions for you, too, which I hope you will answer.

The policies recommended to achieve a significant impact (assuming, magically, that the BRIC countries will go along) are terribly punitive. Too many proponents of reducing CO2 emissions are dishonest about this. BTW, cap and trade is punitive - if it is to have an effect - because it only works by increasing the price of carbon emissions to a level where it is painful - sounds punitive to me.
If I thought reducing CO2 emissions to a level to make a difference (if one believes alarmist predictions) would be anything other than a serious drag on the economy, I would be more willing to go along.

I'm not a fan of cap and trade, either, but let's look at what you are doing here.

First of all, you are implicitly defining ANY policy whatsoever as subjectively "punitive", a priori. Thus your false dichotomy (nothing versus punitive) is artificially made permanent.

But I find this more important -

(if one believes alarmist predictions)

All actual evidence is labeled as "alarmist".

I strongly request a reply to each of these questions -

1) Is there any possible evidence that could convince you that extremely rapid oxydization of fossil fuels by humans has unwanted impact on climate? What would that evidence be?

2) Is there any policy you could ever accept that involved any reduction of CO2 emissions? If so, describe what such a policy would be like.

But in all seriousness, beyond the verbal sparring, you have yet to address the issue of the BRIC countries. Do you advocate that America adopts expensive CO2 mitigation activities if these countries do not? Please be specific.

Let's get rid of the loaded, subjective word "expensive".

Let's also put aside the obvious inaccuracy of your claim that BRIC countries intend to develop oil-dependent economies. Neither Brazil* nor India shows this tendency, at least not relative to the US. *Brazil has some other issues. Let's treat the question as a hypothetical.

The obvious logical answer is "OF COURSE".

Does this make sense - "My neighbor's dog sometimes shits on my lawn, so therefore, I will encourage my own dog to shit all over my lawn as well"? No, it doesn't, because less shit is still better.

Now let me clarify what a good policy would be - it does not resemble current US policy.

It would be grounded in early tax-funded encouragement of less harmful technologies (all of which have some issues of their own, of course), including, at this stage, responsible nuclear power, although true renewable energy like solar, wind, and hydro should be emphasized. As it happens, the US military has current programs to reduce its dependence on foreign oil, for obvious reasons, so much of this could be accomplished by diverting military spending away from negative-marginal-utility projects into energy projects. This would strongly resemble the way the US built railroad, highway, and air travel infrastructure.

This would need to be accompanied by flexible mandates requiring automobiles and appliances sold in the US, buildings constructed in the US, etc, to meet gradually increasing energy efficiency requirements.

My expectation would be that this could be accomplished with little or no impact on the quality of life of the US consumer (or positive impact), and that it would be imitated by developing economies, who have no reason to wish to tie themselves to an anachronistic "cheap foreign oil" model.

That's what a greenhouse gas is, and the most abundant one on Earth is carbon dioxide, or CO2"

Wrong. According to Wikipedia "When these gases are ranked by their contribution to the greenhouse effect, the most important are: water vapor which contributes 36â72%, carbon dioxide which contributes 9â26%, methane which contributes 4â9%, ozone which contributes 3â7%."

So WikiP says that water vapor is the biggest contributor to the greenhouse effect, not CO2. And that does NOT include clouds. When you include cloud's contribution to the greenhouse effect, Wikipedia says "When considering water vapor and clouds together, the contribution is between 66-85%."

So CO2 is relegated to a position far down the list. It is insignificant. And humanity contributes only 3% of the earths total annual CO2 output. In the end, we are nothing to the greenhouse effect.

Klem--by that "reasoning," if all water on the planet were to vanish instantly it would be insignificant, because there had always been and would continue to be more dirt.

The first plot you show is "global average temperature". Can you please explain what that is?

In my first week of thermodynamic class, I was told we cannot average intensive properties (like temperature).

How can you compute the average temperature in a room, for example? Put thermometers all over the place, then add up the readings and divide by number of thermometers? What if one thermometer is in contact with the ice cube in my drink, or resting the electric heater in the corner? Do you weight the readings by the density of the media they are touching? Or their heat capacity? This is why you can't average intensive properties.

I am *not* a global warming skeptic; I simply do not understand what "average global temperature" means, and after a modest amount of searching, I have not found any reference explaining what this term means when used by climatologists.

By WaveRider (not verified) on 17 Nov 2010 #permalink

Klem -

I think you need to ask yourself some questions.

According to Wikipedia "When these gases are ranked by their contribution to the greenhouse effect, the most important are: water vapor which contributes 36â72%, carbon dioxide which contributes 9â26%, methane which contributes 4â9%, ozone which contributes 3â7%."
So WikiP says that water vapor is the biggest contributor to the greenhouse effect, not CO2. And that does NOT include clouds. When you include cloud's contribution to the greenhouse effect, Wikipedia says "When considering water vapor and clouds together, the contribution is between 66-85%."

Okay, this is all reasonable, but where do you think they got that information from? From climatologists. The same guys who discovered this are the guys who are explaining to you that rapid human oxidization of fossil fuels is contributing to a net warming trend.

So CO2 is relegated to a position far down the list. It is insignificant.

Non sequitur. It does not follow from anything you said above that increasing CO2 concentration in the atmosphere will not impact climate.

And humanity contributes only 3% of the earths total annual CO2 output. In the end, we are nothing to the greenhouse effect.

First of all, as I explained above, earth's total CO2 output is mainly part of the carbon cycle, and does not end up causing a long term net increase in atmospheric concentration of CO2.

Now, let me ask you a couple of questions. Don't bother to reply to me without answering the questions.

1) What is the quantitative relationship between atmospheric CO2 concentration and earth's mean surface temperature? How much does the CO2 concentration of the atmosphere need to change to cause some given increase in mean surface temperature of the earth?

2) Is there any evidence that could convince you that human output of CO2 is contributing to a warming effect? What would that evidence be?

It's not a fraud, they all know that it's a problem and something has to be done. It's not like all the world leaders just fancied a wet weekend in Copenhagen. The problem is that to sort out the mess will mean some cuts (which nobody likes) or at least some changes (which nobody likes) and it's all on a national or even continental scale. It's a massive negotiation between rich and poor; developed, developing and undeveloped countries.
http://www.healthproductreviewers.com/leanspa-acai-reviews.html

By Roger Buchanan (not verified) on 22 Nov 2010 #permalink

Water vapor is an output though. Like, if you watch the weather reports, the dew points are high in warm temperatures, and low in cold temperatures. There's only so much water vapor our atmosphere can hold before it rains or snows, yo.

Co2 is underground and shit in petroleum and shit.

The rest of the world is way worse about Global Warming than the US. They have recently decided to change the name to Climate Change due to the most recent winter being one of the coldest on record in decades. The whole thing goes in cycles and I bet we're heading into a cooling period due to the decrease in sun activity recently.
http://brainquickensite.com/

By Jessie Mccauley (not verified) on 26 Nov 2010 #permalink

As a geologist, I think that there is nothing that is happening now that isn't consistent with history. Change is the norm. Those that don't understand that are not geologists. We don't have enought knowledge and technology to know how much humans are contributing to any warming. We certainly don't know the warming is harmful overall.
http://musclexedge.org/

By Michael Gettys (not verified) on 29 Nov 2010 #permalink

There is one easy way to test people believe in and their commitment to fighting global warming and that is to give every person a ration of carbon if you go over or use more than your ration then you pay much much more for your polluting lifestyle.
Money and the ability to pay should NOT repeat NOT be a licence to pollute.
http://mojoblastreview.com

By James Leblanc (not verified) on 03 Dec 2010 #permalink

Global warming's not a fraud as such. It is happening. What is less clear is how much is due to man's activities and how much is simply the earth heating up slightly, as it does. The mid 1600s were warmer than now, and I don't remember Shakespeare writing about Range Rovers!
What bothers many people is the fact that it is being used by our government simply as a way of extracting yet more tax from us.
http://www.wellnessstarts.com/nitric-oxide-rush-reviews.html

By Bruce Ball (not verified) on 06 Dec 2010 #permalink

Well I am grasping at strings here I myself can stop and maybe reverse global warming, if my understanding of the way the planet works my theory for stopping the melting of the polar ice caps might be useful to thoughs of you who have the financial backing to >DO< something. I mean I have ideas to for many problems the world is facing, but being in the currant financial state I am in can do nothing, but remain silent really. This is really my last venue for trying to find someone to hear me out or even point me in the right direction?

Where can I find help to help the world???

I have a lot of possible solutions,
but no way to implement them.

By Crackpot Inventor (not verified) on 22 Mar 2011 #permalink

Is there human-created global warming? I certainly don't know.

But in the article, the argument is made that: "As the data shows in no uncertain terms, the world is — on average — getting warmer. And it’s doing so at a very fast rate: the global average temperature has increased by approximately one degree Celsius in the last century"

A century is a long time for us, but an insanely short time in history - in fact, it is roughly 0.00000002% of the earth's history.

To put it in perspective, if the history of the earth was mapped over the span of a year (1 year = the 4.54 billion that earth has been around up until today), then the 100-years in question would be represented by the last 0.63072 SECONDS of that year.

A blip, a veritable blink or two in the vast history of the planet. To me, that is hardly a large enough sample size to determine a trend, even before we start to look at causation.

Sure, it could turn out to be a trend. It could be an anomaly. It could be part of a temporary spike. Who knows, and it will take far more than the last, or the next 0.00000002% of earth's history to figure that out.

We should definitely reduce emissions and pollution - but to the end of creating a healthier environment to live in, to give us cleaner air to breathe - not necessarily to manipulate a climate that we are egotistical enough to think we are in control of.

By Frank Johnson (not verified) on 22 Oct 2013 #permalink

Hello there! Do you use Twitter? I'd like to follow you if
that would be ok. I'm definitely enjoying your blog
and look forward to new updates.

"Is there human-created global warming? I certainly don’t know."

Ever tried to find out? You know, actually investigate the claims without a priori decisions?

"A century is a long time for us, but an insanely short time in history – in fact, it is roughly 0.00000002% of the earth’s history. "

Apparently not.

Well, that was a great help for my physics project. Thanks a lot!!!

By Raima Ibnat Ch… (not verified) on 30 Apr 2015 #permalink

If we are emitting that much CO2 that is effecting the atmosphere, how can we prevent it? If the CO2 is what's causing "Global Warming" scientists have had to have found some way to prevent the amount of CO2 damaging the earth.

"If we are emitting that much CO2 that is effecting the atmosphere, how can we prevent it?"

By not emitting it.

Duh.

Do you wonder how you can stop getting fat if eating lots of cake is what makes you fat?

"If the CO2 is what’s causing “Global Warming” scientists have had to have found some way to prevent the amount of CO2 damaging the earth."

And its not an "If".

And there's no need (or way) to stop CO2 damaging the earth.

And all we need to do is STOP BURNING FOSSIL FUELS.

How difficult is it to STOP doing something? It requires LESS EFFORT than doing it!

Why is it so hard to work that out for some people?

"Radiation physics constraints on global warming: CO2 increase has little effect"

Well, that was a load of bollocks.

It was solved in the mid 20th century.

Go here to read the history of the science of climate and CO2's effects on it:

https://www.aip.org/history/climate/index.htm